CARDIO MED SURG

Pataasin ang iyong marka sa homework at exams ngayon gamit ang Quizwiz!

2. A er teaching about ways to decrease risk factors for CAD, the nurse recognizes that additional instruction is needed when the patient says a. "I would like to add weight li ing to my exercise program." b. "I can only keep my blood pressure normal with medication." c. "I can change my diet to decrease my intake of saturated fats." d. "I will change my lifestyle to reduce activities that increase mystress."

2. Correct answer: a Rationale: Risk factors for coronary artery disease include elevated serum levels of lipids, elevated blood pressure, tobacco use, physical inactivity, obesity, diabetes, metabolic syndrome, certain psychologic states, and elevated homocysteine levels. Weight lifting is not a cardioprotective exercise. An example of health-promoting regular physical activity is brisk walking (3 to 4 miles/hr) for at least 30 minutes five or more times each week.

2. A compensatory mechanism involved in HF that leads to inappropriate fluid retention and additional workload of the heart is a. ventricular dilation. b. ventricular hypertrophy. c. neurohormonal response. d. sympathetic nervous system activation.

2. Correct answer: c Rationale: The following mechanisms in heart failure lead to inappropriate fluid retention and additional workload of the heart: activation of the renin-angiotensin-aldosterone system (RAAS) cascade and release of antidiuretic hormone from the posterior pituitary gland in response to low cerebral perfusion pressure that results from low cardiac output.

14. In palpating the patient's pedal pulses, the nurse determines the pulses are absent. What factor could contribute to this result? A Atherosclerosis B Hyperthyroidism C Arteriovenous fistula D Cardiac dysrhythmias

A Atherosclerosis can cause an absent peripheral pulse. The feet would also be cool and may be discolored. Hyperthyroidism causes a bounding pulse. Arteriovenous fistula gives a thrill or vibration to the vessel, although this would not be in the foot. Cardiac dysrhythmias cause an irregular pulse rhythm.

7. A patient with a recent diagnosis of heart failure has been prescribed furosemide (Lasix) in an effort to physiologically do what for the patient? A Reduce preload. B Decrease afterload. C Increase contractility. D Promote vasodilation.

A Diuretics such as furosemide are used in the treatment of HF to mobilize edematous fluid, reduce pulmonary venous pressure, and reduce preload. They do not directly influence afterload, contractility, or vessel tone.

2. While admitting a patient with pericarditis, the nurse will assess for what manifestations of this disorder? A Pulsus paradoxus B Prolonged PR intervals C Widened pulse pressure D Clubbing of the fingers

A Pericarditis can lead to cardiac tamponade, an emergency situation. Pulsus paradoxus greater than 10 mm Hg is a sign of cardiac tamponade that should be assessed at least every 4 hours in a patient with pericarditis. Prolonged PR intervals occur with first-degree AV block. Widened pulse pressure occurs with valvular heart disease. Clubbing of fingers may occur in subacute forms of infective endocarditis and valvular heart disease.

When providing dietary instruction to a patient with hypertension, the nurse would advise the patient to restrict intake of which meat? A Broiled fish B Roasted duck Correct C Roasted turkey D Baked chicken breast

B Roasted duck is high in fat, which should be avoided by the patient with hypertension. Weight loss may slow the progress of atherosclerosis and overall CVD risk. The other meats are lower in fat and are therefore acceptable in the diet.

35. The nurse teaches the patient with any venous disorder that the best way to prevent venous stasis and increase venous return is to a. walk. b. sit with the legs elevated. c. frequently rotate the ankles. d. continuously wear elastic compression stockings.

a. During walking, the muscles of the legs continuously knead the veins, promoting movement of venous blood toward the heart. Walking is the best measure to prevent venous stasis. The other methods will help venous return but they do not provide the benefit that ambulation does.

38. When the patient who is diagnosed with an MI is not relieved of chest pain with IV nitroglycerin, which medication will the nurse expect to be used? a. IV morphine sulfate c. IV amiodarone (Cordarone) b. Calcium channel blockers d. Angiotensin-converting enzyme (ACE) inhibitors

a. Morphine sulfate decreases anxiety and cardiac workload as a vasodilator and reduces preload and myocardial O2 consumption, which relieves chest pain. Calcium channel blockers, amiodarone, and ACE inhibitors will not relieve chest pain related to an MI.

8. The patient who is being admitted has had a history of uncontrolled hypertension. High SVR is most likely to cause damage to which organ? a. Brain c. Retina b. Heart d. Kidney

b. The increased systemic vascular resistance (SVR) of hypertension directly increases the workload of the heart and heart failure occurs when the heart can no longer pump effectively against the increased resistance. The heart may be indirectly damaged by atherosclerotic changes in the blood vessels, as are the brain, retina, and kidney.

43. Which statement indicates the patient is experiencing anger as the psychologic response to his acute MI? a. "Yes, I'm having a little chest pain. It's no big deal." b. "I don't think I can take care of myself at home yet." c. "What's going to happen if I have another heart attack?" d. "I hope my wife is happy now after harping at me about my eating habits all these years."

d. Anger about the MI may be directed at family, staff, or the medical regimen. Stating that the chest pain is no big deal is denial. Relaying an inability to care for self relates to dependency. Questioning what will happen if there is another attack is expressing anxiety and fear. Depression may be expressed related to changes in lifestyle. Realistic acceptance is seen with actively engaging in changing modifiable risk factors.

25. A 52-year-old man is admitted to the emergency department with severe chest pain. On what basis would the nurse suspect an MI? a. He has pale, cool, clammy skin. b. He reports nausea and vomited once at home. c. He says he is anxious and has a feeling of impending doom. d. He reports he has had no relief of the pain with rest or position change.

d. The subjective report of the pain from an MI is usually severe. It usually is unrelieved by nitroglycerin, rest, or position change and usually lasts more than the 15 or 20 minutes typical of angina pain. All of the other symptoms may occur with angina as well as with an MI.

11. A patient was admitted to the emergency department (ED) 24 hours earlier with complaints of chest pain that were subsequently attributed to ST-segment-elevation myocardial infarction (STEMI). What complication of MI should the nurse anticipate? A Unstable angina B Cardiac tamponade C Sudden cardiac death D Cardiac dysrhythmias

D The most common complication after MI is dysrhythmias, which are present in 80% of patients. Unstable angina is considered a precursor to MI rather than a complication. Cardiac tamponade is a rare event, and sudden cardiac death is defined as an unexpected death from cardiac causes. Cardiac dysfunction in the period following an MI would not be characterized as sudden cardiac death.

1. The nurse recognizes that primary manifestations of systolic failure include a. ↓ EF and ↑ PAWP. b. ↓ PAWP and ↑ EF. c. ↓ pulmonary hypertension associated with normal EF. d. ↓ a erload and ↓ left ventricular end-diastolic pressure.

1. Correct answer: a Rationale: Systolic heart failure results in systolic failure in the left ventricle (LV). The LV loses its ability to generate enough pressure to eject blood forward through the aorta. This results in increased pulmonary artery wedge pressure (PAWP). The hallmark of systolic failure is a decrease in the left ventricular ejection fraction (EF).

1. A 50-year-old woman weighs 95 kg and has a history of tobacco use, high blood pressure, high sodium intake, and sedentary life- style. When developing an individualized care plan for her, the nurse determines that the most important risk factors for periph- eral artery disease (PAD) that need to be modified are a. weight and diet. b. activity level and diet. c. tobacco use and high blood pressure. d. sedentary lifestyle and high blood pressure.

1. Correct answer: c Rationale: Significant risk factors for peripheral artery disease include tobacco use, hyperlipidemia, elevated levels of high-sensitivity C-reactive protein, diabetes mellitus, and uncontrolled hypertension; the most important is tobacco use. Other risk factors include family history, hypertriglyceridemia, hyperuricemia, increasing age, obesity, sedentary lifestyle, and stress.

A patient with a tricuspid valve disorder will have impaired blood ow between the a. vena cava and right atrium. b. le atrium and le ventricle. c. right atrium and right ventricle. d. right ventricle and pulmonary artery.

1. Correct answer: c Rationale: The tricuspid valve is located between the right atrium and the right ventricle.

1. Assessment of an IV cocaine user with infective endocarditis should focus on which signs and symptoms (select all that apply)? a. Retinal hemorrhages b. Splinter hemorrhages c. Presence of Osler's nodes d. Painless nodules over bony prominences e. Painless erythematous macules on the palms and soles

1. Correct answers: a, b, c, e Rationale: Clinical manifestations of infective endocarditis may include hemorrhagic retinal lesions (Roth's spots), splinter hemorrhages (black, longitudinal streaks) that may occur in the nail beds, Osler's nodes (painful, tender, red or purple, pea-size lesions) on the fingertips or toes, and Janeway's lesions (flat, painless, small, red spots) on the palms and soles.

1. Which BP-regulating mechanism(s) can result in the development of hypertension if defective (select all that apply)? a. Release of norepinephrine b. Secretion of prostaglandins c. Stimulation of the sympathetic nervous system d. Stimulation of the parasympathetic nervous system e. Activation of the renin-angiotensin-aldosterone system

1. Correct answers: a, c, e Rationale: Norepinephrine (NE) is released from the sympathetic nervous system nerve endings and activates receptors located in the vascular smooth muscle. When the α-adrenergic receptors in smooth muscle of the blood vessels are stimulated by NE, vasoconstriction results. Increased sympathetic nervous system stimulation produces increased vasoconstriction and increased renin release. Increased renin levels activate the renin-angiotensin-aldosterone system, leading to elevation in blood pressure (BP).

1. In teaching a patient about coronary artery disease, the nurse explains that the changes that occur in this disorder include (select all that apply) a. diffuse involvement of plaque formation in coronary veins. b. abnormal levels of cholesterol, especially low-density lipoproteins. c. accumulation of lipid and brous tissue within the coronary arteries. d. development of angina due to a decreased blood supply to the heart muscle. e. chronic vasoconstriction of coronary arteries leading to permanent vasospasm.

1. Correct answers: b, c, d Rationale: Atherosclerosis is the major cause of coronary artery disease (CAD) and is characterized by a focal deposit of cholesterol and lipids, primarily within the intimal wall of the artery. The endothelial lining of the coronary arteries becomes inflamed from the presence of unstable plaques and the oxidation of low-density lipoprotein (LDL) cholesterol. Fibrous plaque causes progressive changes in the endothelium of the arterial wall. The result is a narrowing of the vessel lumen and a reduction in blood flow to the myocardial tissue.

10. The nurse is caring for a patient newly admitted with heart failure secondary to dilated cardiomyopathy. Which intervention would be a priority? a. Encourage caregivers to learn CPR. b. Consider a consultation with hospice for palliative care. c. Monitor the patient's response to prescribed medications. d. Arrange for the patient to enter a cardiac rehabilitation program.

10. Correct answer: c Rationale: Observing for signs and symptoms of worsening heart failure, dysrhythmias, and embolus formation in patients with dilated cardiomyopathy is essential, as is monitoring drug responsiveness. The goal of therapy is to keep the patient at an optimal level of functioning and out of the hospital. The priority intervention is to manage the acute symptoms with medications. The caregivers should learn cardiopulmonary resuscitation (CPR) before hospital discharge, and the patient may be referred to cardiac rehabilitation. Patients with dilated cardiomyopathy with progression to class IV stage D heart failure are candidates for palliative care.

10. The recommended treatment for an initial VTE in an otherwise healthy person with no signi cant co-morbidities would include a. IV argatroban (Acova) as an inpatient. b. IV unfractionated heparin as an inpatient. c. subcutaneous unfractionated heparin as an outpatient. d. subcutaneous low-molecular-weight heparin as an outpatient.

10. Correct answer: d Rationale: Patients with confirmed VTE should receive initial treatment with low-molecular-weight heparin (LMWH), unfractionated heparin (UH), fondaparinux, or rivaroxaban, followed by warfarin for 3 months to maintain the international normalized ratio (INR) between 2.0 and 3.0 for 24 hours. Patients with multiple comorbid conditions, complex medical issues, or a very large VTE usually are hospitalized for treatment and typically receive intravenous UH. LMWH only for 3 months is another option for patients with acute VTE. Depending on the clinical presentation, patients often can be managed safely and effectively as outpatients.

10. Which nursing responsibilities are priorities when caring for a patient returning from a cardiac catheterization (select all that apply)? a. Monitoring vital signs and ECG b. Checking the catheter insertion site and distal pulses c. Assisting the patient to ambulate to the bathroom to void d. Informing the patient that he will be sleepy from the general anesthesia e. Instructing the patient about the risks of the radioactive isotope injection

10. Correct answers: a, b Rationale: The nursing responsibilities after cardiac catheterization include assessment of the puncture site for hematoma and bleeding; assessment of circulation to the extremity used for catheter insertion and of peripheral pulses, color, and sensation of the extremity; and monitoring vital signs and electrocardiographic rhythm. Other nursing responsibilities are described in Table 32-6.

2. A 45-year-old patient with chronic arterial disease has a brachial systolic blood pressure (SBP) of 132 mm Hg and an ankle SBP of 102 mm Hg. The ankle-brachial index is______________ and indicates____________ (mild/moderate/severe) arterial disease.

102 ÷ 132 = 0.77; mild (see Table 38-3)

11. A key aspect of teaching for the patient on anticoagulant therapy includes which instructions? a. Monitor for and report any signs of bleeding. b. Do not take acetaminophen (Tylenol) for a headache. c. Decrease your dietary intake of foods containing vitamin K. d. Arrange to have blood drawn routinely to check drug levels.

11. Correct answer: a Rationale: Patients taking anticoagulants should be taught to monitor and report any signs of bleeding, which can be a serious complication. Other important patient teaching includes maintenance of a consistent intake of foods containing vitamin K, avoidance of supplements that contain vitamin K, and routine coagulation laboratory studies if a patient is taking warfarin.

12. In planning care and patient teaching for the patient with venous leg ulcers, the nurse recognizes that the most important intervention in healing and control of this condition is a. sclerotherapy. b. using moist environment dressings. c. taking horse chestnut seed extract daily. d. applying elastic compression stockings.

12. Correct answer: d Rationale: Compression is essential for treating chronic venous insufficiency (CVI), healing venous ulcers, and preventing ulcer recurrence. Use of custom-fitted elastic compression stockings is one option for compression therapy.

2. While obtaining subjective assessment data from a patient with hypertension, the nurse recognizes that a modi able risk factor for the development of hypertension is a. a low-calcium diet. b. excessive alcohol consumption. c. a family history of hypertension. d. consumption of a high-protein diet.

2. Correct answer: b Rationale: Alcohol intake is a modifiable risk factor for hypertension. Excessive alcohol intake is strongly associated with hypertension. Patients with hypertension should limit their daily intake to 1 oz of alcohol.

2. The nurse is caring for a patient with chronic constrictive pericarditis. Which assessment finding reflects a more serious complication of this condition? a. Fatigue b. Peripheral edema c. Jugular venous distention d. Thickened pericardium on echocardiography

2. Correct answer: c Rationale: Cardiac tamponade is a serious complication of pericarditis. As the compression of the heart increases, decreased left atrial filling results in decreases in cardiac output. Neck veins usually are markedly distended as a result of jugular venous pressure elevation related to compression of the right side of the heart.

2. Rest pain is a manifestation of PAD that occurs due to a chronic a. vasospasm of small cutaneous arteries in the feet. b. increase in retrograde venous blood ow in the legs. c. decrease in arterial blood ow to the nerves of the feet. d. decrease in arterial blood ow to the leg muscles during exercise.

2. Correct answer: c Rationale: Rest pain most often occurs in the forefoot or toes and is aggravated by limb elevation. Rest pain occurs when blood flow is insufficient to meet basic metabolic requirements of the distal tissues. Rest pain occurs more often at night because cardiac output tends to drop during sleep and the limbs are at the level of the heart. Patients often try to achieve partial pain relief by dangling the leg over the side of the bed or sleeping in a chair to allow gravity to maximize blood flow.

2. A patient has a severe blockage in his right coronary artery. Which cardiac structures are most likely to be a ected by this blockage (select all that apply)? a. AV node b. Le ventricle c. Coronary sinus d. Right ventricle e. Pulmonic valve

2. Correct answers: a, b, d Rationale: The right coronary artery (RCA) supplies blood to the right atrium, the right ventricle, and a portion of the posterior wall of the left ventricle. In 90% of people, the RCA supplies blood to the atrioventricular (AV) node, the bundle of His, and part of the cardiac conduction system.

3. A hospitalized patient with a history of chronic stable angina tells the nurse that she is having chest pain. e nurse bases his actions on the knowledge that ischemia a. will always progress to myocardial infarction. b. will be relieved by rest, nitroglycerin, or both. c. indicates that irreversible myocardial damage is occurring. d. is frequently associated with vomiting and extreme fatigue.

3. Correct answer: b Rationale: Chronic stable angina is chest pain that occurs intermittently over a long period with the same pattern of onset, duration, and intensity of symptoms. The chest pain is relieved by rest or by rest and medication (e.g., nitroglycerin). The ischemia is transient and does not cause myocardial damage.

3. The portion of the vascular system responsible for hemostasis is the a. thin capillary vessels. b. endothelial layer of the arteries. c. elastic middle layer of the veins. d. smooth muscle of the arterial wall.

3. Correct answer: b Rationale: The innermost lining of the arteries is the endothelium. The endothelium maintains hemostasis, promotes blood flow, and under normal conditions, inhibits blood coagulation.

3. A patient with infective endocarditis develops sudden le leg pain with pallor, paresthesia, and a loss of peripheral pulses. The nurse's initial action should be to a. elevate the leg to promote venous return. b. start anticoagulant therapy with IV heparin. c. notify the physician of the change in peripheral perfusion. d. place the bed in reverse Trendelenburg to promote perfusion.

3. Correct answer: c Rationale: The patient has potentially developed acute arterial ischemia (sudden interruption in the arterial blood supply to the extremity), caused by an embolism from a cardiac thrombus that occurred as a complication of infective endocarditis. Clinical manifestations of acute arterial ischemia include any or all of the six Ps : pain, pallor, paralysis, pulselessness, paresthesia, and poikilothermia. Without immediate intervention, ischemia may progress quickly to tissue necrosis and gangrene within a few hours. If the nurse detects these signs, the physician should be notified immediately.

3. In teaching a patient with hypertension about controlling the con- dition, the nurse recognizes that a. all patients with elevated BP require medication. b. obese persons must achieve a normal weight to lower BP. c. it is not necessary to limit salt in the diet if taking a diuretic. d. lifestyle modi cations are indicated for all persons with elevated BP.

3. Correct answer: d Rationale: Lifestyle modifications are indicated for all patients with prehypertension and hypertension.

3. A patient is admitted with myocarditis. While performing the initial assessment, the nurse may find which clinical signs and symptoms (select all that apply)? a. Angina b. Pleuritic chest pain c. Splinter hemorrhages d. Pericardial friction rub e. Presence of Osler's nodes

3. Correct answers: a, b, d Rationale: Clinical manifestations of myocarditis may include early systemic manifestations (i.e., fever, fatigue, malaise, myalgias, pharyngitis, dyspnea, lymphadenopathy, and nausea and vomiting), early cardiac manifestations (e.g., pleuritic chest pain with a pericardial friction rub and effusion), and late cardiac signs (e.g., S3 heart sound, crackles, jugular venous distention [JVD], syncope, peripheral edema, and angina).

3. You are caring for a patient with ADHF who is receiving IV dobutamine (Dobutrex). You know that this drug is ordered because it (select all that apply) a. increases SVR. b. produces diuresis. c. improves contractility. d. dilates renal blood vessels. e. works on the β1-receptors in the heart.

3. Correct answers: c, e Rationale: Dobutamine (Dobutrex) has a positive chronotropic effect and increases heart rate and improves contractility. It is a selective β-adrenergic agonist and works primarily on the β1-adrenergic receptors in the heart. It is frequently used in the short-term management of acute decompensated heart failure (ADHF).

4. When a person's blood pressure rises, the homeostatic mechanism to compensate for an elevation involves stimulation of a. baroreceptors that inhibit the sympathetic nervous system, causing vasodilation. b. chemoreceptors that inhibit the sympathetic nervous system, causing vasodilation. c. baroreceptors that inhibit the parasympathetic nervous system, causing vasodilation. d. chemoreceptors that stimulate the sympathetic nervous system, causing an increased heart rate.

4. Correct answer: a Rationale: Baroreceptors in the aortic arch and carotid sinus are sensitive to stretch or pressure within the arterial system. Stimulation of these receptors sends information to the vasomotor center in the brainstem. This results in temporary inhibition of the sympathetic nervous system and enhancement of the parasympathetic influence, which cause a decrease in heart rate and peripheral vasodilation.

4. The nurse is caring for a patient who is 2 days post-MI. e patient reports that she is experiencing chest pain. She states, "It hurts when I take a deep breath." Which action would be a priority? a. Notify the physician STAT and obtain a 12-lead ECG. b. Obtain vital signs and auscultate for a pericardial friction rub. c. Apply high- ow oxygen by face mask and auscultate breath sounds. d. Medicate the patient with PRN analgesic and reevaluate in 30 minutes.

4. Correct answer: b Rationale: Acute pericarditis is inflammation of the visceral and/or parietal pericardium; it often occurs 2 to 3 days after an acute myocardial infarction. Chest pain may vary from mild to severe and is aggravated by inspiration, coughing, and movement of the upper body. Sitting in a forward position often relieves the pain. The pain is usually different from pain associated with a myocardial infarction. Assessment of the patient with pericarditis may reveal a friction rub over the pericardium.

4. A major consideration in the management of the older adult with hypertension is to a. prevent primary hypertension from converting to secondary hypertension. b. recognize that the older adult is less likely to adhere to the drug therapy regimen than a younger adult. c. ensure that the patient receives larger initial doses of antihypertensive drugs because of impaired absorption. d. use careful technique in assessing the BP of the patient because of the possible presence of an auscultatory gap.

4. Correct answer: d Rationale: Careful technique is important in assessing BP in older adults. In some older people, there is a wide gap between the first Korotkoff sound and subsequent beats; such a wide interval is called an auscultatory gap. Failure to inflate the cuff high enough may result in a serious underestimate of systolic BP.

4. Priority nursing management for a patient with myocarditis includes interventions related to a. meticulous skin care. b. antibiotic prophylaxis. c. tight glycemic control. d. oxygenation and ventilation.

4. Correct answer: d Rationale: General supportive measures for management of myocarditis include interventions to improve ventilation and oxygenation (oxygen therapy, bed rest, and restricted activity).

4. A patient with chronic HF and atrial brillation is treated with a digitalis glycoside and a loop diuretic. To prevent possible compli- cations of this combination of drugs, what does the nurse need to do (select all that apply)? a. Monitor serum potassium levels. b. Teach the patient how to take a pulse rate. c. Keep an accurate measure of intake and output. d. Teach the patient about dietary restriction of potassium. e. Withhold digitalis and notify health care provider if heart rate is irregular.

4. Correct answers: a, b Rationale: Hypokalemia, which can be caused by the use of potassium-depleting diuretics (e.g., thiazides, loop diuretics), is one of the most common causes of digitalis toxicity. Low serum levels of potassium enhance the action of digitalis, causing a therapeutic dose to achieve toxic levels. Hypokalemia can also precipitate dysrhythmias. Monitoring the serum potassium levels of patients receiving digitalis preparations and potassium-depleting diuretics is essential. Patients taking digitalis preparations should be taught how to measure their pulse rate because bradycardia and atrioventricular blocks are late signs of digitalis toxicity. In addition, patients should know what pulse rate would necessitate a call to the health care provider.

4. Which clinical manifestations are seen in patients with either Buerger's disease or Raynaud's phenomenon (select all that apply)? a. Intermittent fevers b. Sensitivity to cold temperatures c. Gangrenous ulcers on ngertips d. Color changes of ngers and toes e. Episodes of super cial vein thrombosis

4. Correct answers: b, c, d Rationale: Both Buerger's disease and Raynaud's phenomenon have the following clinical manifestations in common: cold sensitivity, ischemic and gangrenous ulcers on fingertips, and color changes of the distal extremity (fingers or toes).

5. When teaching a patient about the long-term consequences of rheumatic fever, the nurse should discuss the possibility of a. valvular heart disease. b. pulmonary hypertension. c. superior vena cava syndrome. d. hypertrophy of the right ventricle.

5. Correct answer: a Rationale: Rheumatic heart disease is a chronic condition resulting from rheumatic fever that is characterized by scarring and deformity of the heart valves.

5. A patient with newly discovered high BP has an average reading of 158/98 mm Hg a er 3 months of exercise and diet modi cations. Which management strategy will be a priority for this patient? a. Medication will be required because the BP is still not at goal. b. BP monitoring should continue for another 3 months to confirm a diagnosis of hypertension. c. Lifestyle changes are less important, since they were not effective, and medications will be started. d. More vigorous changes in the patient's lifestyle are needed for a longer time before starting medications.

5. Correct answer: a Rationale: The patient has hypertension, stage 1. Lifestyle modifications will continue, but drug initiation of therapy is a priority. Reduction of BP can help to prevent serious complications related to hypertension.

5. A patient is admitted to the hospital with a diagnosis of abdominal aortic aneurysm. Which signs and symptoms would suggest that his aneurysm has ruptured? a. Sudden shortness of breath and hemoptysis b. Sudden, severe low back pain and bruising along his ank c. Gradually increasing substernal chest pain and diaphoresis d. Sudden, patchy blue mottling on feet and toes and rest pain

5. Correct answer: b Rationale: The clinical manifestations of a ruptured abdominal aortic aneurysm include severe back pain, back or flank ecchymosis (Grey Turner's sign), and hypovolemic shock (tachycardia, hypotension, pale clammy skin, decreased urine output, altered level of consciousness, and abdominal tenderness).

5. A P wave on an ECG represents an impulse arising at the a. SA node and repolarizing the atria. b. SA node and depolarizing the atria. c. AV node and depolarizing the atria. d. AV node and spreading to the bundle of His.

5. Correct answer: b Rationale: The first wave, P, begins with the firing of the sinoatrial (SA) node and represents depolarization of the fibers of the atria

5. A patient is admitted to the ICU with a diagnosis of unstable angina. Which medication(s) would the nurse expect the patient to receive (select all that apply)? a. ACE inhibitor b. Antiplatelet therapy c. rombolytic therapy d. Prophylactic antibiotics e. Intravenous nitroglycerin

5. Correct answers: a, b, e Rationale: In addition to oxygen, several medications may be used to treat unstable angina (UA): nitroglycerin, aspirin (chewable), and morphine. For patients with UA with negative cardiac markers and ongoing angina, a combination of aspirin, heparin, and a glycoprotein IIb/IIIa inhibitor (e.g., eptifibatide [Integrilin]) is recommended. Angiotensin-converting enzyme (ACE) inhibitors decrease myocardial oxygen demand by producing vasodilation, reducing blood volume, and slowing or reversing cardiac remodeling.

5. Patients with a heart transplantation are at risk for which complications in the rst year a er transplantation (select all that apply)? a. Cancer b. Infection c. Rejection d. Vasculopathy e. Sudden cardiac death

5. Correct answers: b, c, e Rationale: A variety of complications can occur after heart transplantation. In the first year after transplantation, the major causes of death are acute rejection and infection. Heart transplant recipients also are at risk for sudden cardiac death. Later, malignancy (especially lymphoma) and cardiac vasculopathy (accelerated CAD) are major causes of death.

6. Which is a priority nursing intervention for a patient during the acute phase of rheumatic fever? a. Administration of antibiotics as ordered b. Management of pain with opioid analgesics c. Encouragement of uid intake for hydration d. Performance of frequent active range-of-motion exercises

6. Correct answer: a Rationale: The primary goal of managing a patient with acute rheumatic fever is to control and eradicate the infecting organism. The nurse should administer antibiotics as ordered to treat the streptococcal infection and teach the patient that completion of the full course of antibiotic therapy is essential for successful treatment.

6. A patient is recovering from an uncomplicated MI. Which rehabilitation guideline is a priority to include in the teaching plan? a. Refrain from sexual activity for a minimum of 3 weeks. b. Plan a diet program that aims for a 1- to 2-pound weight loss per week. c. Begin an exercise program that aims for at least ve 30-minutesessions per week. d. Consider the use of erectile agents and prophylactic NTG before engaging in sexual activity.

6. Correct answer: c Rationale: Physical activity should be regular, rhythmic, and repetitive, with the use of large muscles to build up endurance (e.g., walking, cycling, swimming, rowing). Physical activity sessions should be at least 30 minutes long. Instruct the patient to begin slowly at personal tolerance (perhaps only 5 to 10 minutes) and build up to 30 minutes.

6. Priority nursing measures a er an abdominal aortic aneurysm repair include a. assessment of cranial nerves and mental status. b. administration of IV heparin and monitoring of aPTT. c. administration of IV uids and monitoring of kidney function. d. elevation of the legs and application of elastic compression stockings.

6. Correct answer: c Rationale: Postoperative priorities include administration of IV fluids and maintenance of renal perfusion. An adequate blood pressure is important for maintaining graft patency, and administration of IV fluids and blood components (as indicated) is essential for adequate blood flow. The nurse should evaluate renal function by measuring hourly urine output and monitoring daily blood urea nitrogen (BUN) and serum creatinine levels. Irreversible renal failure may occur after aortic surgery, particularly in individuals at high risk.

6. A patient is admitted to the hospital in hypertensive emergency (BP 244/142 mm Hg). Sodium nitroprusside is started to treat the ele- vated BP. Which management strategy(ies) would be appropriate for this patient (select all that apply)? a. Measuring hourly urine output b. Decreasing the MAP by 50% within the rst hour c. Continuous BP monitoring with an intraarterial line d. Maintaining bed rest and providing tranquilizers to lower the BP e. Assessing the patient for signs and symptoms of heart failure and changes in mental status

6. Correct answers: a, c, e Rationale: Measure urine output hourly to assess renal perfusion. Patients being treated with IV sodium nitroprusside should have continuous intraarterial BP monitoring. Hypertensive crisis can cause encephalopathy, intracranial or subarachnoid hemorrhage, acute left ventricular failure, myocardial infarction, renal failure, dissecting aortic aneurysm, and retinopathy. The initial treatment goal is to decrease the mean atrial pressure (MAP) by no more than 25% within minutes to 1 hour. Patients receiving IV antihypertensive drugs may be restricted to bed rest. Getting up (e.g., to use the toilet) may cause severe cerebral ischemia and fainting.

6. When collecting subjective data related to the cardiovascular system, which information should be obtained from the patient (select all that apply)? a. Annual income b. Smoking history c. Religious preference d. Number of pillows used to sleep e. Blood for basic laboratory studies

6. Correct answers: b, c, d Rationale: The health history should include assessment of tobacco use. The patient should be asked about any cultural or religious beliefs that may influence the management of the cardiovascular problem. Patients with heart failure may need to sleep with the head elevated on pillows or sleep in a chair.

7. The first priority of collaborative care of a patient with a suspected acute aortic dissection is to a. reduce anxiety. b. control blood pressure. c. monitor for chest pain. d. increase myocardial contractility.

7. Correct answer: b Rationale: The initial goals of therapy for acute aortic dissection without complications are blood pressure (BP) control and pain management. BP control reduces stress on the aortic wall by reducing systolic BP and myocardial contractility.

7. e auscultatory area in the le midclavicular line at the level of the h ICS is the best location to hear sounds from which heart valve? a. Aortic b. Mitral c. Tricuspid d. Pulmonic

7. Correct answer: b Rationale: The mitral valve can be assessed by auscultation at the left midclavicular line at the fifth intercostal space (ICS).

7. e most common nding in individuals at risk for sudden cardiac death is a. aortic valve disease. b. mitral valve disease. c. left ventricular dysfunction. d. atherosclerotic heart disease.

7. Correct answer: c Rationale: Left ventricular dysfunction (ejection fraction less than 30%) and ventricular dysrhythmias after myocardial infarction are the strongest predictors of sudden cardiac death (SCD)

7. Which clinical finding would most likely indicate decreased cardiac output in a patient with aortic valve regurgitation? a. Reduction in peripheral edema and weight b. Carotid venous distention and new-onset atrial brillation c. Signi cant pulsus paradoxus and diminished peripheral pulses d. Shortness of breath on minimal exertion and a diastolic murmur

7. Correct answer: d Rationale: Clinical manifestations of aortic regurgitation (AR) that indicate decreased cardiac output include severe dyspnea, chest pain, and hypotension. Other manifestations of chronic AR include water-hammer pulse (i.e., a strong, quick beat that collapses immediately), soft or absent S1, presence of S3 or S4, and soft, high-pitched diastolic murmur. A low-pitched diastolic murmur may be heard in severe AR. Early manifestations may include exertional dyspnea, orthopnea, and paroxysmal nocturnal dyspnea.

8. When assessing a patient, you note a pulse de cit of 23 beats. is nding may be caused by a. dysrhythmias. b. heart murmurs. c. gallop rhythms. d. pericardial friction rubs.

8. Correct answer: a Rationale: A pulse deficit occurs if there is a difference between the apical and radial beats per minute. A pulse deficit indicates cardiac dysrhythmias.

8. The patient at highest risk for venous thromboembolism (VTE) is a. a 62-year-old man with spider veins who is having arthroscopic knee surgery. b. a 32-year-old woman who smokes, takes oral contraceptives, and is planning a trip to Europe. c. a 26-year-old woman who is 3 days postpartum and received maintenance IV uids for 12 hours during her labor. d. an active 72-year-old man at home recovering from transurethral resection of the prostate for benign prostatic hyperplasia.

8. Correct answer: b Rationale: Three important factors (called Virchow's triad) in the etiology of venous thrombosis are (1) venous stasis, (2) damage of the endothelium (inner lining of the vein), and (3) hypercoagulability of the blood. Patients at risk for venous thrombosis usually have predisposing conditions for these three disorders (see Table 38-8). The 32-year-old woman has the highest risk: long trips without adequate exercise (venous stasis), tobacco use, and use of oral contraceptives. Note: The likelihood of hypercoagulability of blood is increased in women older than 35 years who use tobacco.

8. A patient is diagnosed with mitral stenosis and new-onset atrial fibrillation. Which interventions could the nurse delegate to unlicensed assistive personnel (UAP) (select all that apply)? a. Obtain and record daily weight. b. Determine apical-radial pulse rate. c. Observe for overt signs of bleeding. d. Obtain and record vital signs, including pulse oximetry. e. Teach the patient how to purchase a Medic Alert bracelet.

8. Correct answers: a, c, d Rationale: The nurse may delegate routine procedures such as measuring weights and vital signs. The nurse may give specific directions to the unlicensed assistive personnel (UAP) to observe and report obvious signs of bleeding. The nurse cannot delegate teaching, assessment, or activities that require clinical judgment. Obtaining an apical-radial pulse rate is an assessment.

9. When assessing the cardiovascular system of a 79-year-old patient, you might expect to find a. a narrowed pulse pressure. b. diminished carotid artery pulses. c. di culty in isolating the apical pulse. d. an increased heart rate in response to stress.

9. Correct answer: c Rationale: Myocardial hypertrophy and the downward displacement of the heart in an older adult may cause difficulty in isolating the apical pulse.

9. Which are probable clinical ndings in a person with an acute lower extremity VTE (select all that apply)? a. Pallor and coolness of foot and calf b. Mild to moderate calf pain and tenderness c. Grossly diminished or absent pedal pulses d. Unilateral edema and induration of the thigh e. Palpable cord along a superficial varicose vein

9. Correct answers: b, d Rationale: The patient with lower extremity venous thromboembolism (VTE) may or may not have unilateral leg edema, extremity pain, a sense of fullness in the thigh or calf, paresthesias, warm skin, erythema, or a systemic temperature greater than 100.4 F (38 C). If the calf is involved, it may be tender to palpation.

Despite a high dosage, a male patient who is taking nifedipine (Procardia XL) for antihypertensive therapy continues to have blood pressures over 140/90 mmHg. What should the nurse do next? A Assess his adherence to therapy. B Ask him to make an exercise plan. C Instruct him to use the DASH diet. D Request a prescription for a thiazide diuretic.

A A long-acting calcium-channel blocker such as nifedipine causes vascular smooth muscle relaxation resulting in decreased SVR and arterial BP and related side effects. The patient data the nurse has about this patient is very limited, so the nurse needs to assess his adherence to therapy.

The nurse is caring for a patient with hypertension who is scheduled to receive a dose of esmolol (Brevibloc). The nurse should withhold the dose and consult the prescribing physician for which vital sign taken just before administration? A Pulse 48 B Respirations 24 C Blood pressure 118/74 D Oxygen saturation 93%

A Because esmolol is a β1-adrenergic blocking agent, it can cause hypotension and bradycardia as adverse effects. The nurse should withhold the dose and consult with the health care provider for parameters regarding pulse rate limits.

18. A 40-year-old man tells the nurse he has a diagnosis for the color and temperature changes of his limbs but can't remember the name of it. He says he must stop smoking and avoid trauma and exposure of his limbs to cold temperatures to get better. This description should allow the nurse to ask the patient if he has which diagnosis? A Buerger's disease B Venous thrombosis C Acute arterial ischemia D Raynaud's phenomenon

A Buerger's disease is a nonatherosclerotic, segmental, recurrent inflammatory disorder of small and medium-sized veins and arteries of upper and lower extremities leading to color and temperature changes of the limbs, intermittent claudication, rest pain, and ischemic ulcerations. It primarily occurs in men younger than 45 years old with a long history of tobacco and/or marijuana use. Buerger's disease treatment includes smoking cessation, trauma and cold temperature avoidance, and a walking program. Venous thrombosis is the formation of a thrombus in association with inflammation of the vein. Acute arterial ischemia is a sudden interruption in arterial blood flow to a tissue caused by embolism, thrombosis, or trauma. Raynaud's phenomenon is characterized by vasospasm-induced color changes of the fingers, toes, ears, and nose.

5. The nurse is providing care for a patient who has decreased cardiac output related to heart failure. What should the nurse recognize about cardiac output? A Calculated by multiplying the patient's stroke volume by the heart rate B The average amount of blood ejected during one complete cardiac cycle C Determined by measuring the electrical activity of the heart and the patient's heart rate D The patient's average resting heart rate multiplied by the patient's mean arterial blood pressure

A Cardiac output is determined by multiplying the patient's stroke volume by heart rate, thus identifying how much blood is pumped by the heart over a 1-minute period. Electrical activity of the heart and blood pressure are not direct components of cardiac output.

1. While doing an admission assessment, the nurse notes clubbing of the patient's fingers. Based on this finding, the nurse will question the patient about which disease process? A Endocarditis B Acute kidney injury C Myocardial infarction D Chronic thrombophlebitis

A Clubbing of the fingers is a loss of the normal angle between the base of the nail and the skin. This finding can be found in endocarditis, congenital defects, and/or prolonged oxygen deficiency. Clinical manifestations of acute kidney injury, myocardial infarction, and chronic thrombophlebitis will not include clubbing of the fingers.

3. The nurse is admitting a 68-year-old preoperative patient with a suspected abdominal aortic aneurysm (AAA). The medication history reveals that the patient has been taking warfarin (Coumadin) on a daily basis. Based on this history and the patient's admission diagnosis, the nurse should prepare to administer which medication? A Vitamin K B Cobalamin C Heparin sodium D Protamine sulfate

A Coumadin is a Vitamin K antagonist anticoagulant that could cause excessive bleeding during surgery if clotting times are not corrected before surgery. For this reason, vitamin K is given as the antidote for warfarin (Coumadin).

1.The nurse teaches a patient with hypertension that uncontrolled hypertension may damage organs in the body primarily by which mechanism? A Hypertension promotes atherosclerosis and damage to the walls of the arteries. B Hypertension causes direct pressure on organs, resulting in necrosis and replacement of cells with scar tissue. C Hypertension causes thickening of the capillary membranes, leading to hypoxia of organ systems. Incorrect D Hypertension increases blood viscosity, which contributes to intravascular coagulation and tissue necrosis distal to occlusions.

A Hypertension is a major risk factor for the development of atherosclerosis by mechanisms not yet fully known. However, once atherosclerosis develops, it damages the walls of arteries and reduces circulation to target organs and tissues.

5. A 64-year-old patient with a history of myocardial infarction is scheduled for a transesophageal echocardiogram to visualize a suspected clot in the left atrium. What information should the nurse include when teaching the patient about this diagnostic study? A IV sedation may be administered to help the patient relax. B Food and fluids are restricted for 2 hours before the procedure. C Ambulation is restricted for up to 6 hours before the procedure. D Contrast medium is injected into the esophagus to enhance images.

A IV sedation is administered to help the patient relax and ease the insertion of the tube into the esophagus. Food and fluids are restricted for at least 6 hours before the procedure. Smoking and exercise are restricted for 3 hours before exercise or stress testing, but not before TEE. Contrast medium is administered IV to evaluate the direction of blood flow if a septal defect is suspected.

23. The patient has CVI and a venous ulcer. The unlicensed assistive personnel (UAP) decides to apply compression stockings because that is what these patients always have ordered. What assessment by the nurse would cause the application of compression stockings to harm the patient? A Rest pain B High blood pressure C Elevated blood sugar D Dry, itchy, flaky skin

A Rest pain occurs as peripheral artery disease (PAD) progresses and involves multiple arterial segments. Compression stockings should not be used on patients with PAD. Elevated blood glucose, possibly indicating uncontrolled diabetes mellitus, and hypertension may or may not indicate arterial problems. Dry, itchy, flaky skin indicates venous insufficiency. The RN should be the one to obtain the order and instruct the UAP to apply compression stockings if they are ordered.

2. Which individuals would the nurse identify as having the highest risk for CAD? A A 45-year-old depressed male with a high-stress job B A 60-year-old male with below normal homocysteine levels C A 54-year-old female vegetarian with elevated high-density lipoprotein (HDL) levels D A 62-year-old female who has a sedentary lifestyle and body mass index (BMI) of 23 kg/m2

A The 45-year-old depressed male with a high-stress job is at the highest risk for CAD. Studies demonstrate that depression and stressful states can contribute to the development of CAD. Elevated HDL levels and low homocysteine levels actually help to prevent CAD. Although a sedentary lifestyle is a risk factor, a BMI of 23 kg/m2 depicts normal weight, and thus the patient with two risk factors is at greatest risk for developing CAD.

7. When planning emergent care for a patient with a suspected MI, what should the nurse anticipate administrating? A Oxygen, nitroglycerin, aspirin, and morphine B Oxygen, furosemide (Lasix), nitroglycerin, and meperidine C Aspirin, nitroprusside (Nipride), dopamine (Intropin), and oxygen D Nitroglycerin, lorazepam (Ativan), oxygen, and warfarin (Coumadin)

A The American Heart Association's guidelines for emergency care of the patient with chest pain include the administration of oxygen, nitroglycerin, aspirin, and morphine. These interventions serve to relieve chest pain, improve oxygenation, decrease myocardial workload, and prevent further platelet aggregation. The other medications may be used later in the patient's treatment.

21. When the patient is being examined for venous thromboembolism (VTE) in the calf, what diagnostic test should the nurse expect to teach the patient about first? A Duplex ultrasound B Contrast venography C Magnetic resonance venography D Computed tomography venography

A The duplex ultrasound is the most widely used test to diagnose VTE. Contrast venography is rarely used now. Magnetic resonance venography is less accurate for calf veins than pelvic and proximal veins. Computed tomography venography may be used but is invasive and much more expensive than the duplex ultrasound.

13. The patient with chronic heart failure is being discharged from the hospital. What information should the nurse emphasize in the patient's discharge teaching to prevent progression of the disease to ADHF? A Take medications as prescribed. B Use oxygen when feeling short of breath. C Only ask the physician's office questions. D Encourage most activity in the morning when rested.

A The goal for the patient with chronic HF is to avoid exacerbations and hospitalization. Taking the medications as prescribed along with nondrug therapies such as alternating activity with rest will help the patient meet this goal. If the patient needs to use oxygen at home, it will probably be used all the time or with activity to prevent respiratory acidosis. Many HF patients are monitored by a care manager or in a transitional program to assess the patient for medication effectiveness and monitor for patient deterioration and encourage the patient. This nurse manager can be asked questions or can contact the health care provider if there is evidence of worsening HF.

1. The community health nurse is planning health promotion teaching targeted at preventing coronary artery disease (CAD). Which ethnic group would the nurse select as the highest priority for this intervention? A White male B Hispanic male C African American male D Native American female

A The incidence of CAD and myocardial infarction (MI) is highest among white, middle-aged men. Hispanic individuals have lower rates of CAD than non-Hispanic whites or African Americans. African Americans have an earlier age of onset and more severe CAD than whites and more than twice the mortality rate of whites of the same age. Native Americans have increased mortality in less than 35-year-olds and have major modifiable risk factors such as diabetes.

1. Which person should the nurse identify as having the highest risk for abdominal aortic aneurysm? A A 70-year-old male, with high cholesterol and hypertension B A 40-year-old female with obesity and metabolic syndrome C A 60-year-old male with renal insufficiency who is physically inactive D A 65-year-old female with hyperhomocysteinemia and substance abuse

A The most common etiology of descending abdominal aortic aneurysm (AAA) is atherosclerosis. Male gender, age 65 years or older, and tobacco use are the major risk factors for AAAs of atherosclerotic origin. Other risk factors include the presence of coronary or peripheral artery disease, high blood pressure, and high cholesterol.

13. Assessment of a patient's peripheral IV site reveals that phlebitis has developed over the past several hours. Which intervention should the nurse implement first? A Remove the patient's IV catheter. B Apply an ice pack to the affected area. C Decrease the IV rate to 20 to 30 mL/hr. D Administer prophylactic anticoagulants.

A The priority intervention for superficial phlebitis is removal of the offending IV catheter. Decreasing the IV rate is insufficient. Anticoagulants are not normally required, and warm, moist heat is often therapeutic.

5. The nurse is reviewing the laboratory test results for a 68-year-old patient whose warfarin (Coumadin) therapy was terminated during the preoperative period. The nurse concludes that the patient is in the most stable condition for surgery after noting which INR (international normalized ratio) result? A 1.0 B 1.8 C 2.7 D 3.4

A The therapeutic range for INR is 2.0 to 3.0 for many clinical diagnoses. The larger the INR number, the greater the amount of anticoagulation. For this reason, the safest value before surgery is 1.0, meaning that the anticoagulation has been reversed.

4. When caring for a patient with infective endocarditis, the nurse will assess the patient for which vascular manifestations (select all that apply)? A Osler's nodes B Janeway's lesions C Splinter hemorrhages D Subcutaneous nodules E Erythema marginatum lesions

A B C Osler's nodes, Janeway's lesions, and splinter hemorrhages are all vascular manifestations of infective endocarditis. Subcutaneous nodules and erythema marginatum lesions occur with rheumatic fever.

3. When providing nutritional counseling for patients at risk for CAD, which foods would the nurse encourage patients to include in their diet (select all that apply)? A Tofu B Walnuts C Tuna fish D Whole milk E Orange juice

A B C Tuna fish, tofu, and walnuts are all rich in omega-3 fatty acids, which have been shown to reduce the risks associated with CAD when consumed regularly.

4. A patient admitted with heart failure appears very anxious and complains of shortness of breath. Which nursing actions would be appropriate to alleviate this patient's anxiety (select all that apply)? A Administer ordered morphine sulfate. B Position patient in a semi-Fowler's position. C Position patient on left side with head of bed flat. D Instruct patient on the use of relaxation techniques. E Use a calm, reassuring approach while talking to patient.

A B D E Morphine sulfate reduces anxiety and may assist in reducing dyspnea. The patient should be positioned in semi-Fowler's position to improve ventilation that will reduce anxiety. Relaxation techniques and a calm reassuring approach will also serve to reduce anxiety.

6. Which effects of aging on the cardiovascular system should the nurse anticipate when providing care for older adults (select all that apply)? A Systolic murmur B Diminished pedal pulses C Increased maximal heart rate D Decreased maximal heart rate E Increased recovery time from activity

A B D E Well-documented cardiovascular effects of the aging process include valvular rigidity leading to systolic murmur, arterial stiffening leading to diminished pedal pulses or possible increased blood pressure, and an increased amount of time that is required for recovery from activity. Maximal heart rate tends to decrease rather than increase with age related to cellular aging and fibrosis of the conduction system.

2. A nurse is caring for a patient immediately following a transesophageal echocardiogram (TEE). Which assessments are appropriate for this patient (select all that apply)? A Assess for return of gag reflex. B Assess groin for hematoma or bleeding. C Monitor vital signs and oxygen saturation. D Position patient supine with head of bed flat. E Assess lower extremities for circulatory compromise.

A C The patient undergoing a TEE has been given conscious sedation and has had the throat numbed with a local anesthetic spray, thus eliminating the gag reflex until the effects wear off. Therefore it is imperative that the nurse assess for gag reflex return before allowing the patient to eat or drink. Vital signs and oxygen saturation are also important assessment parameters resulting from the use of sedation. A TEE does not involve invasive procedures of the circulatory blood vessels. Therefore it is not necessary to monitor the patient's groin and lower extremities in relation to this procedure or to maintain a flat position.

10. The patient had myocarditis and is now experiencing fatigue, weakness, palpitations, and dyspnea at rest. The nurse assesses pulmonary crackles, edema, and weak peripheral pulses. Sinoatrial tachycardia is evident on the cardiac monitor. The Doppler echocardiography shows dilated cardiomyopathy. What collaborative and nursing care of this patient should be done to improve cardiac output and the quality of life? (Select all that apply.) A Decrease preload and afterload. B Relieve left ventricular outflow obstruction. C Control heart failure by enhancing myocardial contractility. D Improve diastolic filling and the underlying disease process. E Improve ventricular filling by reducing ventricular contractility.

A C The patient is experiencing dilated cardiomyopathy. To improve cardiac output and quality of life, drug, nutrition, and cardiac rehabilitation will be focused on controlling heart failure by decreasing preload and afterload and improving cardiac output, which will improve the quality of life. Relief of left ventricular outflow obstruction and improving ventricular filling by reducing ventricular contractility is done for hypertrophic cardiomyopathy. There are no specific treatments for restrictive cardiomyopathy, but interventions are aimed at improving diastolic filling and the underlying disease process.

15. What medications should the nurse expect to include in the teaching plan to decrease the risk of cardiovascular events and death for PAD patients (select all that apply)? A Ramipril (Altace) B Cilostazol (Pletal) C Simvastatin (Zocor) D Clopidogrel (Plavix) E Warfarin (Coumadin) F Aspirin (acetylsalicylic acid)

A C F Angiotensin-converting enzyme inhibitors (e.g., ramipril [Altace]) are used to control hypertension. Statins (e.g., simvastatin [Zocor]) are used for lipid management. Aspirin is used as an antiplatelet agent. Cilostazol (Pletal) is used for intermittent claudication, but it does not reduce CVD morbidity and mortality risks. Clopidogrel may be used if the patient cannot tolerate aspirin. Anticoagulants (e.g., warfarin [Coumadin]) are not recommended to prevent CVD events in PAD patients.

10. The patient has heart failure (HF) with an ejection fraction of less than 40%. What core measures should the nurse expect to include in the plan of care for this patient (select all that apply)? A Left ventricular function is documented. B Controlling dysrhythmias will eliminate HF. C Prescription for digoxin (Lanoxin) at discharge D Prescription for angiotensin-converting enzyme (ACE) inhibitor at discharge E Education materials about activity, medications, weight monitoring, and what to do if symptoms worsen

A D E The Joint Commission has identified these three core measures for heart failure patients. Although controlling dysrhythmias will improve CO and workload, it will not eliminate HF. Prescribing digoxin for all HF patients is no longer done because there are newer effective drugs and digoxin toxicity occurs easily related to electrolyte levels and the therapeutic range must be maintained.

Modifiable risk factors associated with CAD include: A. age, weight, and cholesterol level. B. smoking, diet, and blood pressure. C. family history, weight, and blood pressure. D. blood glucose level, activity level, and family history.

Answer: B. Smoking, diet, and blood pressure are modifiable risk factors; age and family history aren't.

A primary goal in the treatment of MI is to: A. prevent blood loss. B. decrease blood pressure. C. relieve pain. D. administer I.V. fluids.

Answer: C. The primary goals in the treatment of MI are to re- lieve pain, stabilize heart rhythm, revascularize the coronary ar- tery, preserve myocardial tissue, and reduce cardiac workload.

The test that's most specific for myocardial damage is: A. CK. B. CK-MB. C. troponin I. D. myoglobin.

Answer: C. Troponin is a protein found in skeletal and cardiac muscles. However, troponin I is found only in the myocardium; it's more specific to myocardial damage than the other choices.

One sign of arterial occlusive disease is: A. a bounding pulse. B. abdominal pain. C. high blood pressure. D. intermittent claudication.

Answer: D. Intermittent claudication is a sign of arterial occlu- sive disease.

5. A 73-year-old man with dementia has a venous ulcer related to chronic venous insufficiency. The nurse should provide education on which type of diet for this patient and his caregiver? A Low-fat diet B High-protein diet C Calorie-restricted diet D High-carbohydrate diet

B A patient with a venous ulcer should have a balanced diet with adequate protein, calories, and micronutrients; this type of diet is essential for healing. Nutrients most important for healing include protein, vitamins A and C, and zinc. Foods high in protein (e.g., meat, beans, cheese, tofu), vitamin A (green leafy vegetables), vitamin C (citrus fruits, tomatoes, cantaloupe), and zinc (meat, seafood) must be provided. Restricting fat or calories is not helpful for wound healing or in patients of normal weight. For overweight individuals with no active venous ulcer, a weight-loss diet should be considered.

3. The nurse is assessing a 62-year-old woman undergoing radiation treatment for breast cancer. How should the nurse position the patient to auscultate for signs of acute pericarditis? A Supine without a pillow B Sitting and leaning forward C Left lateral sidelying position D Head of bed at a 45-degree angle

B A pericardial friction rub indicates pericariditis. To auscultate a pericardial friction rub, the patient should be sitting and leaning forward. The nurse will hear the pericardial friction rub at the end of expiration.

3. The nurse conducts a complete physical assessment on a patient admitted with infective endocarditis. Which finding is significant? A Respiratory rate of 18 and heart rate of 90 B Regurgitant murmur at the mitral valve area C Heart rate of 94 and capillary refill time of 2 seconds D Point of maximal impulse palpable in fourth intercostal space

B A regurgitant murmur of the aortic or mitral valves would indicate valvular disease, which is a complication of endocarditis. All the other findings are within normal limits.

9. The nurse is providing teaching to a patient recovering from an MI. How should resumption of sexual activity be discussed? A Delegated to the primary care provider B Discussed along with other physical activities C Avoided because it is embarrassing to the patient D Accomplished by providing the patient with written material

B Although some nurses may not feel comfortable discussing sexual activity with patients, it is a necessary component of patient teaching. It is helpful to consider sex as a physical activity and to discuss or explore feelings in this area when other physical activities are discussed. Although providing the patient with written material is appropriate, it should not replace a verbal dialogue that can address the individual patient's questions and concerns.

5. The nurse admits a 73-year-old male patient with dementia for treatment of uncontrolled hypertension. The nurse will closely monitor for hypokalemia if the patient receives which medication? A Clonidine (Catapres) B Bumetanide (Bumex) C Amiloride (Midamor) D Spironolactone (Aldactone)

B Bumetanide is a loop diuretic. Hypokalemia is a common adverse effect of this medication. Amiloride is a potassium-sparing diuretic. Spironolactone is an aldosterone-receptor blocker. Hyperkalemia is an adverse effect of both amiloride and spironolactone. Clonidine is a central-acting α-adrenergic antagonist and does not cause electrolyte abnormalities.

6. What should the nurse recognize as an indication for the use of dopamine (Intropin) in the care of a patient with heart failure? A Acute anxiety B Hypotension and tachycardia C Peripheral edema and weight gain D Paroxysmal nocturnal dyspnea (PND)

B Dopamine is a β-adrenergic agonist whose inotropic action is used for treatment of severe heart failure accompanied by hemodynamic instability. Such a state may be indicated by tachycardia accompanied by hypotension. PND, anxiety, edema, and weight gain are common signs and symptoms of heart failure, but these do not necessarily warrant the use of dopamine.

1. A nurse is caring for a patient with a diagnosis of deep venous thrombosis (DVT). The patient has an order to receive 30 mg enoxaparin (Lovenox). Which injection site should the nurse use to administer this medication safely? A Buttock, upper outer quadrant B Abdomen, anterior-lateral aspect C Back of the arm, 2 inches away from a mole D Anterolateral thigh, with no scar tissue nearby

B Enoxaparin (Lovenox) is a low-molecular-weight (LMW) heparin that is given as a deep subcutaneous injection in the right and left anterolateral abdomen. All subcutaneous injections should be given away from scars, lesions, or moles.

Roasted duck is high in fat, which should be avoided by the patient with hypertension. Weight loss may slow the progress of atherosclerosis and overall CVD risk. The other meats are lower in fat and are therefore acceptable in the diet. A Weight loss of 2 lb B Blood pressure 128/86 C Absence of ankle edema D Output of 600 mL per 8 hours

B Hydrochlorothiazide may be used alone as monotherapy to manage hypertension or in combination with other medications if not effective alone. After the first few weeks of therapy, the diuretic effect diminishes, but the antihypertensive effect remains. Since the patient has been taking this medication for 10 years, the most direct measurement of its intended effect would be the blood pressure.

2. A 67-year-old man with peripheral artery disease is seen in the primary care clinic. Which symptom reported by the patient would indicate to the nurse that the patient is experiencing intermittent claudication? A Patient complains of chest pain with strenuous activity. B Patient says muscle leg pain occurs with continued exercise. C Patient has numbness and tingling of all his toes and both feet. D Patient states the feet become red if he puts them in a dependent position.

B Intermittent claudication is an ischemic muscle ache or pain that is precipitated by a consistent level of exercise, resolves within 10 minutes or less with rest, and is reproducible. Angina is the term used to describe chest pain with exertion. Paresthesia is the term used to describe numbness or tingling in the toes or feet. Reactive hyperemia is the term used to describe redness of the foot; if the limb is in a dependent position the term is dependent rubor.

14. A 62-year-old Hispanic male patient with diabetes mellitus has been diagnosed with peripheral artery disease (PAD). The patient is a smoker and has a history of gout. What should the nurse focus her teaching on to prevent complications for this patient? A Gender B Smoking C Ethnicity D Co-morbidities

B Smoking is the most significant factor for this patient. PAD is a marker of advanced systemic atherosclerosis. Therefore tobacco cessation is essential to reduce PAD progression, CVD events, and mortality. Diabetes mellitus and hyperuricemia are also risk factors. Being male or Hispanic are not risk factors for PAD.

11. The patient is confused about how there can be a blockage in the left anterior descending artery (LAD), but there is damage to the right ventricle. The nurse can help the patient understand this with which explanation? A "The one vessel curves around from the left side to the right ventricle." B "The LAD supplies blood to the left side of the heart and part of the right ventricle." C "The right ventricle is supplied during systole primarily by the right coronary artery." D "It is actually on your right side of the heart, but we call it the left anterior descending vessel."

B The best response is explaining that the lower portion of the right ventricle receives blood flow from the left anterior descending artery as well as the right coronary artery during diastole.

2. The nurse supervises an unlicensed assistant personnel (UAP) who is taking the blood pressure of 58-year-old female patient admitted with heart failure. The patient is obese. The nurse should intervene if what is observed? A The UAP waits 2 minutes after position changes to take orthostatic pressures. B The UAP deflates the blood pressure cuff at a rate of 8 to 10 mm Hg per second. C The UAP takes the blood pressure with the patient's arm at the level of the heart. D The UAP takes a forearm blood pressure because the largest cuff will not fit the patient's upper arm.

B The cuff should be deflated at a rate of 2 to 3 mm Hg per second. The arm should be supported at the level of the heart for accurate blood pressure measurements. If the maximum size blood pressure cuff does not fit the upper arm, the forearm may be used. Orthostatic blood pressures should be taken within 1 to 2 minutes of repositioning the patient.

2. The nurse is preparing to administer digoxin to a patient with heart failure. In preparation, laboratory results are reviewed with the following findings: sodium 139 mEq/L, potassium 5.6 mEq/L, chloride 103 mEq/L, and glucose 106 mg/dL. What should the nurse do next? A Withhold the daily dose until the following day. B Withhold the dose and report the potassium level. C Give the digoxin with a salty snack, such as crackers. D Give the digoxin with extra fluids to dilute the sodium level.

B The normal potassium level is 3.5 to 5.0 mEq/L. The patient is hyperkalemic, which makes the patient more prone to digoxin toxicity. For this reason, the nurse should withhold the dose and report the potassium level. The physician may order the digoxin to be given once the potassium level has been treated and decreases to within normal range.

10. The nurse is caring for a newly admitted patient with vascular insufficiency. The patient has a new order for enoxaparin (Lovenox) 30 mg subcutaneously. What should the nurse do to correctly administer this medication? A Spread the skin before inserting the needle. B Leave the air bubble in the prefilled syringe. C Use the back of the arm as the preferred site. D Sit the patient at a 30-degree angle before administration.

B The nurse should not expel the air bubble from the prefilled syringe because it should be injected to clear the needle of medication and avoid leaving medication in the needle track in the tissue.

14. The patient comes to the ED with severe, prolonged angina that is not immediately reversible. The nurse knows that if the patient once had angina related to a stable atherosclerotic plaque and the plaque ruptures, there may be occlusion of a coronary vessel and this type of pain. How will the nurse document this situation related to pathophysiology, presentation, diagnosis, prognosis, and interventions for this disorder? A Unstable angina B Acute coronary syndrome (ACS) C ST-segment-elevation myocardial infarction (STEMI) D Non-ST-segment-elevation myocardial infarction (NSTEMI)

B The pain with ACS is severe, prolonged, and not easy to relieve. ACS is associated with deterioration of a once-stable atherosclerotic plaque that ruptures, exposes the intima to blood, and stimulates platelet aggregation and local vasoconstriction with thrombus formation. The unstable lesion, if partially occlusive, will be manifest as unstable angina or NSTEMI. If there is total occlusion, it is manifest as a STEMI.

2. When teaching a patient about dietary management of stage 1 hypertension, which instruction is most appropriate? A Restrict all caffeine. B Restrict sodium intake. C Increase protein intake. D Use calcium supplements.

B The patient should decrease intake of sodium. This will help to control hypertension, which can be aggravated by excessive salt intake, which in turn leads to fluid retention. Caffeine and protein intake do not affect hypertension. Calcium supplements are not recommended to lower BP.

8. The patient had a history of rheumatic fever and has been diagnosed with mitral valve stenosis. The patient is planning to have a biologic valve replacement. What protective mechanisms should the nurse teach the patient about using after the valve replacement? A Long-term anticoagulation therapy B Antibiotic prophylaxis for dental care C Exercise plan to increase cardiac tolerance D Take β-adrenergic blockers to control palpitations.

B The patient will need to use antibiotic prophylaxis for dental care to prevent endocarditis. Long-term anticoagulation therapy is not used with biologic valve replacement unless the patient has atrial fibrillation. An exercise plan to increase cardiac tolerance is needed for a patient with heart failure. Taking β-adrenergic blockers to control palpitations is prescribed for mitral valve prolapse, not valve replacement.

12. A patient with varicose veins has been prescribed compression stockings. How should the nurse teach the patient to use these? A "Try to keep your stockings on 24 hours a day, as much as possible." B "While you're still lying in bed in the morning, put on your stockings." C "Dangle your feet at your bedside for 5 minutes before putting on your stockings." D "Your stockings will be most effective if you can remove them for a few minutes several times a day."

B The patient with varicose veins should apply stockings in bed, before rising in the morning. Stockings should not be worn continuously, but they should not be removed several times daily. Dangling at the bedside prior to application is likely to decrease their effectiveness.

17. A patient was just diagnosed with acute arterial ischemia in the left leg secondary to atrial fibrillation. Which early clinical manifestation must be reported to the physician immediately to save the patient's limb? A Paralysis B Paresthesia C Crampiness D Referred pain

B The physician must be notified immediately if any of the six Ps of acute arterial ischemia occur to prevent ischemia from quickly progressing to tissue necrosis and gangrene. The six Ps are paresthesia, pain, pallor, pulselessness, and poikilothermia, with paralysis being a very late sign indicating the death of nerves to the extremity. Crampy leg sensation is more common with varicose veins. The pain is not referred.

3. The nurse is admitting a patient who is scheduled to undergo a cardiac catheterization. What allergy is most important for the nurse to assess before this procedure? A Iron B Iodine C Aspirin D Penicillin

B The physician will usually use an iodine-based contrast to perform this procedure. Therefore it is imperative to know whether or not the patient is allergic to iodine or shellfish. Knowledge of allergies to iron, aspirin, or penicillin will be secondary.

12. The nurse is examining the ECG of a patient who has just been admitted with a suspected MI. Which ECG change is most indicative of prolonged or complete coronary occlusion? A Sinus tachycardia B Pathologic Q wave C Fibrillatory P waves D Prolonged PR interval

B The presence of a pathologic Q wave, as often accompanies STEMI, is indicative of complete coronary occlusion. Sinus tachycardia, fibrillatory P waves (e.g., atrial fibrillation), or a prolonged PR interval (first-degree heart block) are not direct indicators of extensive occlusion.

20. The patient had aortic aneurysm repair. What priority nursing action will the nurse use to maintain graft patency? A Assess output for renal dysfunction. B Use IV fluids to maintain adequate BP. C Use oral antihypertensives to maintain cardiac output. D Maintain a low BP to prevent pressure on surgical site

B The priority is to maintain an adequate BP (determined by the surgeon) to maintain graft patency. A prolonged low BP may result in graft thrombosis, and hypertension may cause undue stress on arterial anastomoses resulting in leakage of blood or rupture at the suture lines, which is when IV antihypertensives may be used. Renal output will be assessed when the aneurysm repair is above the renal arteries to assess graft patency, not maintain it.

5. After teaching a patient with chronic stable angina about nitroglycerin, the nurse recognizes the need for further teaching when the patient makes which statement? A "I will replace my nitroglycerin supply every 6 months." B "I can take up to five tablets every 3 minutes for relief of my chest pain." C "I will take acetaminophen (Tylenol) to treat the headache caused by nitroglycerin." D "I will take the nitroglycerin 10 minutes before planned activity that usually causes chest pain."

B The recommended dose of nitroglycerin is one tablet taken sublingually (SL) or one metered spray for symptoms of angina. If symptoms are unchanged or worse after 5 minutes, the patient should be instructed to activate the emergency medical services (EMS) system. If symptoms are improved, repeat the nitroglycerin every 5 minutes for a maximum of three doses and contact EMS if symptoms have not resolved completely.

10. While auscultating the patient's heart sounds with the bell of the stethoscope, the nurse hears these sounds. How should the nurse document what is heard? A Diastolic murmur B Third heart sound (S3) C Fourth heart sound (S4) D Normal heart sounds (S1, S2)

B The third heart sound is heard closely after the S2 and is known as a ventricular gallop because it is a vibration of the ventricular walls associated with decreased compliance of the ventricles during filling. It occurs with left ventricular failure. Murmurs sound like turbulence between normal heart sounds and are caused by abnormal blood flow through diseased valves. The S4 heart sound is a vibration caused by atrial contraction, precedes the S1, and is known as an atrial gallop. The normal S1 and S2 are heard when the valves close normally.

1. A 55-year-old man with aortic valve stenosis is being admitted for valve replacement surgery. Which assessment finding should the nurse expect? A Pulse deficit B Systolic murmur C Distended neck veins D Splinter hemorrhages

B The turbulent blood flow across a diseased valve results in a murmur. Aortic stenosis produces a systolic murmur. A pulse deficit indicates a cardiac dysrhythmia, most commonly atrial fibrillation. Distended neck veins may be caused by right-sided heart failure. Splinter hemorrhages occur in patients with infective endocarditis.

8. A patient with a diagnosis of heart failure has been started on a nitroglycerin patch by his primary care provider. What should this patient be taught to avoid? A High-potassium foods B Drugs to treat erectile dysfunction C Nonsteroidal antiinflammatory drugs D Over-the-counter H2-receptor blockers

B The use of erectile drugs concurrent with nitrates creates a risk of severe hypotension and possibly death. High-potassium foods, NSAIDs, and H2-receptor blockers do not pose a risk in combination with nitrates.

16. A female patient with critical limb ischemia has had peripheral artery bypass surgery to improve her circulation. What care should the nurse provide on postoperative day 1? A Keep the patient on bed rest. B Assist the patient with walking several times. C Have the patient sit in the chair several times. D Place the patient on her side with knees flexed.

B To avoid blockage of the graft or stent, the patient should walk several times on postoperative day 1 and subsequent days. Having the patient's knees flexed for sitting in a chair or in bed increase the risk of venous thrombosis and may place stress on the suture lines.

9. A 59-year-old man has presented to the emergency department with chest pain. What component of his subsequent blood work is most clearly indicative of a myocardial infarction (MI)? A CK-MB B Troponin C Myoglobin D C-reactive protein

B Troponin is the biomarker of choice in the diagnosis of MI, with sensitivity and specificity that exceed those of CK-MB and myoglobin. CRP levels are not used to diagnose acute MI.

7. Auscultation of a patient's heart reveals the presence of a murmur. What is this assessment finding a result of? A Increased viscosity of the patient's blood B Turbulent blood flow across a heart valve C Friction between the heart and the myocardium D A deficit in heart conductivity that impairs normal contractility

B Turbulent blood flow across the affected valve results in a murmur. A murmur is not a direct result of variances in blood viscosity, conductivity, or friction between the heart and myocardium.

When assessing the patient for orthostatic hypotension, after taking the blood pressure (BP) and pulse (P) in the supine position, what should the nurse do next? A Repeat BP and P in this position. B Take BP and P with patient sitting. C Record the BP and P measurements. D Take BP and P with patient standing.

B When assessing for orthostatic changes in BP after measuring BP in the supine position, the patient is placed in a sitting position and BP is measured within 1 to 2 minutes and then repositioned to the standing position with BP measured again, within 1 to 2 minutes. The results are then recorded with a decrease of 20 mm Hg or more in SBP, a decrease of 10 mm Hg or more in DBP, and/or an increase in pulse of greater than or equal to 20 beats/minute from supine to standing indicating orthostatic hypotension.

4. A nurse is preparing to teach a group of women in a community volunteer group about heart disease. What should the nurse include in the teaching plan? A Women are less likely to delay seeking treatment than men. B Women are more likely to have noncardiac symptoms of heart disease. C Women are often less ill when presenting for treatment of heart disease. D Women experience more symptoms of heart disease at a younger age than men.

B Women often have atypical angina symptoms and nonpain symptoms. Women experience the onset of heart disease about 10 years later than men. Women are often more ill on presentation and delay longer in seeking care than men.

6. The nurse would assess a patient with complaints of chest pain for which clinical manifestations associated with a myocardial infarction (MI) (select all that apply)? A Flushing B Ashen skin C Diaphoresis D Nausea and vomiting E S3 or S4 heart sounds

B C D E During the initial phase of an MI, catecholamines are released from the ischemic myocardial cells, causing increased sympathetic nervous system (SNS) stimulation. This results in the release of glycogen, diaphoresis, and vasoconstriction of peripheral blood vessels. The patient's skin may be ashen, cool, and clammy (not flushed) as a result of this response. Nausea and vomiting may result from reflex stimulation of the vomiting center by severe pain. Ventricular dysfunction resulting from the MI may lead to the presence of the abnormal S3 and S4 heart sounds.

4. Which antilipemic medications should the nurse question for a patient with cirrhosis of the liver (select all that apply)? A Niacin (Nicobid) B Ezetimibe (Zetia) C Gemfibrozil (Lopid) D Atorvastatin (Lipitor) E Cholestyramine (Questran)

B D Ezetimibe (Zetia) should not be used by patients with liver impairment. Adverse effects of atorvastatin (Lipitor), a statin drug, include liver damage and myopathy. Liver enzymes must be monitored frequently and the medication stopped if these enzymes increase. Niacin's side effects subside with time, although decreased liver function may occur with high doses. Cholestyramine is safe for long-term use.

3. What is the priority assessment by the nurse caring for a patient receiving IV nesiritide (Natrecor) to treat heart failure? A Urine output B Lung sounds C Blood pressure D Respiratory rate

C Although all identified assessments are appropriate for a patient receiving IV nesiritide, the priority assessment would be monitoring for hypotension, the main adverse effect of nesiritide.

The nurse is caring for a patient admitted with chronic obstructive pulmonary disease (COPD), angina, and hypertension. Before administering the prescribed daily dose of atenolol 100 mg PO, the nurse assesses the patient carefully. Which adverse effect is this patient at risk for, given the patient's health history? A Hypocapnia B Tachycardia C Bronchospasm D Nausea and vomiting

C Atenolol is a cardioselective β1-adrenergic blocker that reduces blood pressure and could affect the β2-receptors in the lungs with larger doses or with drug accumulation. Although the risk of bronchospasm is less with cardioselective β-blockers than nonselective β-blockers, atenolol should be used cautiously in patients with COPD.

6. The nurse would determine that a postoperative patient is not receiving the beneficial effects of enoxaparin (Lovenox) after noting what during a routine shift assessment? A Generalized weakness and fatigue B Crackles bilaterally in the lung bases C Pain and swelling in lower extremity D Abdominal pain with decreased bowel sounds

C Enoxaparin is a low-molecular-weight heparin used to prevent the development of deep vein thromboses (DVTs) in the postoperative period. Pain and swelling in the lower extremity can indicate development of DVT and therefore may signal ineffective medication therapy.

8. A postoperative patient asks the nurse why the physician ordered daily administration of enoxaparin (Lovenox). Which reply by the nurse is most appropriate? A "This medication will help prevent breathing problems after surgery, such as pneumonia." B "This medication will help lower your blood pressure to a safer level, which is very important after surgery." C "This medication will help prevent blood clots from forming in your legs until your level of activity, such as walking, returns to normal." D "This medication is a narcotic pain medication that will help take away any muscle aches caused by positioning on the operating room table."

C Enoxaparin is an anticoagulant that is used to prevent DVTs postoperatively. All other explanations/options do not describe the action/purpose of enoxaparin.

8. While assessing the cardiovascular status of a patient, the nurse performs auscultation. Which intervention should the nurse implement in the assessment during auscultation? A Position the patient supine. B Ask the patient to hold his or her breath. C Palpate the radial pulse while auscultating the apical pulse. D Use the bell of the stethoscope when auscultating S1 and S2.

C In order to detect a pulse deficit, simultaneously palpate the radial pulse when auscultating the apical area. The diaphragm is more appropriate than the bell when auscultating S1 and S2. A sitting or side-lying position is most appropriate for cardiac auscultation. It is not necessary to ask the patient to hold his or her breath during cardiac auscultation.

When teaching how lisinopril (Zestril) will help lower the patient's blood pressure, which mechanism of action should the nurse use to explain it? A Blocks β-adrenergic effects. B Relaxes arterial and venous smooth muscle. C Inhibits conversion of angiotensin I to angiotensin II. D Reduces sympathetic outflow from central nervous system.

C Lisinopril is an angiotensin-converting enzyme (ACE) inhibitor that inhibits the conversion of angiotensin I to angiotensin II, which reduces angiotensin II-mediated vasoconstriction and sodium and water retention. Beta blockers result in vasodilation and decreased heart rate. Direct vasodilators relax arterial and venous smooth muscle. Central acting α-adrenergic antagonists reduce sympathetic outflow from the CNS to produce vasodilation and decreased SVR and BP.

1. The nurse teaches a 28-year-old man newly diagnosed with hypertension about lifestyle modifications to reduce his blood pressure. Which statement by the patient requires an intervention by the nurse? A "I will avoid adding salt to my food during or after cooking." B "If I lose weight, I might not need to continue taking medications." C "I can lower my blood pressure by switching to smokeless tobacco." D "Diet changes can be as effective as taking blood pressure medications."

C Nicotine contained in tobacco products (smoking and chew) cause vasoconstriction and increase blood pressure. Persons with hypertension should restrict sodium to 1500 mg/day by avoiding foods high in sodium and not adding salt in preparation of food or at meals. Weight loss can decrease blood pressure between 5 to 20 mm Hg. Following dietary recommendations (such as the DASH diet) lowers blood pressure, and these decreases compare with those achieved with blood pressure-lowering medication.

5. What nursing action should the nurse prioritize during the care of a patient who has recently recovered from rheumatic fever? A Teach the patient how to manage his or her physical activity. B Teach the patient about the need for ongoing anticoagulation. C Teach the patient about the need for continuous antibiotic prophylaxis. D Teach the patient about the need to maintain standard infection control procedures.

C Patients with a history of rheumatic fever frequently require ongoing antibiotic prophylaxis, an intervention that necessitates education. This consideration is more important than activity management in preventing recurrence. Anticoagulation is not indicated in this patient population. Standard precautions are indicated for all patients.

10. Postoperative care of a patient undergoing coronary artery bypass graft (CABG) surgery includes monitoring for what common complication? A Dehydration B Paralytic ileus C Atrial dysrhythmias D Acute respiratory distress syndrome

C Postoperative dysrhythmias, specifically atrial dysrhythmias, are common in the first 3 days following CABG surgery. Although the other complications could occur, they are not common complications.

3. A 32-year-old female is prescribed diltiazem (Cardizem) for Raynaud's phenomenon. To evaluate the patient's expected response to this medication, what is most important for the nurse to assess? A Improved skin turgor B Decreased cardiac rate C Improved finger perfusion D Decreased mean arterial pressure

C Raynaud's phenomenon is an episodic vasospastic disorder of small cutaneous arteries, most frequently involving the fingers and toes. Diltiazem (Cardizem) is a calcium channel blocker that relaxes smooth muscles of the arterioles by blocking the influx of calcium into the cells, thus reducing the frequency and severity of vasospastic attacks. Perfusion to the fingertips is improved and vasospastic attacks reduced. Diltiazem may decrease heart rate and blood pressure, but that is not the purpose in Raynaud's phenomenon. Skin turgor is most often a reflection of hydration status.

7. A 25-year-old patient with a group A streptococcal pharyngitis does not want to take the antibiotics prescribed. What should the nurse tell the patient to encourage the patient to take the medications and avoid complications of the infection? A "The complications of this infection will affect the skin, hair, and balance." B "You will not feel well if you do not take the medicine and get over this infection." C "Without treatment, you could get rheumatic fever, which can lead to rheumatic heart disease." D "You may not want to take the antibiotics for this infection, but you will be sorry if you do not."

C Rheumatic fever (RF) is not common because of effective use of antibiotics to treat streptococcal infections. Without treatment, RF can occur and lead to rheumatic heart disease, especially in young adults. The complications do not include hair or balance. Saying that the patient will not feel well or that the patient will be sorry if the antibiotics are not taken is threatening to the patient and inappropriate for the nurse to say.

13. When looking at the electrocardiogram (ECG) of the patient, the nurse knows that the QRS complex recorded on the ECG represents which part of the heart's beat? A Depolarization of the atria B Repolarization of the ventricles C Depolarization from AV node throughout ventricles D The length of time it takes for the impulse to travel from the atria to the ventricles

C The QRS recorded on the ECG represents depolarization from the AV node throughout the ventricles. The P wave represents depolarization of the atria. The T wave represents repolarization of the ventricles. The interval between the PR and QRS represents the length of time it takes for the impulse to travel from the atria to the ventricles.

20. The nurse assesses the right femoral artery puncture site as soon as the patient arrives after having a stent inserted into a coronary artery. The insertion site is not bleeding or discolored. What should the nurse do next to ensure the femoral artery is intact? A Palpate the insertion site for induration. B Assess peripheral pulses in the right leg. C Inspect the patient's right side and back. D Compare the color of the left and right legs.

C The best method to determine that the right femoral artery is intact after inspection of the insertion site is to logroll the patient to inspect the right side and back for retroperitoneal bleeding. The artery can be leaking and blood is drawn into the tissues by gravity. The peripheral pulses, color, and sensation of the right leg will be assessed per agency protocol.

4. A 67-year-old woman with a history of coronary artery disease and prior myocardial infarction is admitted to the emergency department with a blood pressure of 234/148 mm Hg and started on IV nitroprusside (Nitropress). What should the nurse determine as an appropriate goal for the first hour of treatment? A Mean arterial pressure lower than 70 mm Hg B Mean arterial pressure no more than 120 mm Hg C Mean arterial pressure no lower than 133 mm Hg D Mean arterial pressure between 70 and 110 mm Hg

C The initial treatment goal is to decrease mean arterial pressure by no more than 25% within minutes to 1 hour. If the patient is stable, the goal for BP is 160/100 to 110 mm Hg over the next 2 to 6 hours. Lowering the blood pressure too much may decrease cerebral, coronary, or renal perfusion and could precipitate a stroke, myocardial infarction, or renal failure. Additional gradual reductions toward a normal blood pressure should be implemented over the next 24 to 48 hours if the patient is clinically stable.

In caring for a patient admitted with poorly controlled hypertension, which laboratory test result should the nurse understand as indicating the presence of target organ damage? A BUN of 15 mg/dL B Serum uric acid of 3.8 mg/dL C Serum creatinine of 2.6 mg/dL D Serum potassium of 3.5 mEq/L

C The normal serum creatinine level is 0.6-1.3 mg/dL. This elevated level indicates target organ damage to the kidneys. The other lab results are within normal limits.

The patient has chronic hypertension. Today she has gone to the ED, and her blood pressure has risen to 200/140. What is the priority assessment for the nurse to make? A Is the patient pregnant? B Does the patient need to urinate? C Does the patient have a headache or confusion? D Is the patient taking antiseizure medications as prescribed?

C The nurse's priority assessments include neurologic deficits, retinal damage, heart failure, pulmonary edema, and renal failure. The headache or confusion could be seen with hypertensive encephalopathy from increased cerebral capillary permeability leading to cerebral edema. Pregnancy can lead to secondary hypertension. Needing to urinate and taking antiseizure medication do not support a hypertensive emergency.

15. In caring for the patient with angina, the patient said, "I walked to the bathroom. While I was having a bowel movement, I started having the worst chest pain ever, like before I was admitted. I called for a nurse, but the pain is gone now." What further assessment data should the nurse obtain from the patient? A "What precipitated the pain?" B "Has the pain changed this time?" C "In what areas did you feel this pain?" D "Rate the pain on a scale from 0 to 10, with 0 being no pain and 10 being the worst pain you can imagine."

C Using PQRST, the assessment data not volunteered by the patient is the radiation of pain, the area the patient felt the pain, and if it radiated. The precipitating event was going to the bathroom and having a bowel movement. The quality of the pain was "like before I was admitted," although a more specific description may be helpful. Severity of the pain was the "worst chest pain ever," although an actual number may be needed. Timing is supplied by the patient describing when the pain occurred and that he had previously had this pain.

9. The nurse is caring for a preoperative patient who has an order for vitamin K by subcutaneous injection. The nurse should verify that which laboratory study is abnormal before administering the dose? A Hematocrit (Hct) B Hemoglobin (Hgb) C Prothrombin time (PT) D Partial thromboplastin time (PTT)

C Vitamin K counteracts hypoprothrombinemia and/or reverses the effects of warfarin (Coumadin) and thus decreases the risk of bleeding. High values for either the prothrombin time (PT) or the international normalized ratio (INR) demonstrates the need for this medication.

4. The nurse is caring for a patient who has been receiving warfarin (Coumadin) and digoxin (Lanoxin) as treatment for atrial fibrillation. Because the warfarin has been discontinued before surgery, the nurse should diligently assess the patient for which complication early in the postoperative period until the medication is resumed? A Decreased cardiac output B Increased blood pressure C Cerebral or pulmonary emboli D Excessive bleeding from incision or IV sites

C Warfarin is an anticoagulant that is used to prevent thrombi from forming on the walls of the atria during atrial fibrillation. Once the medication is terminated, thrombi could again form. If one or more thrombi detach from the atrial wall, they could travel as cerebral emboli from the left atrium or pulmonary emboli from the right atrium.

The nurse is teaching a women's group about prevention of hypertension. What information should be included in the teaching for all the women (select all that apply)? Lose weight. Limit nuts and seeds. Limit sodium and fat intake. Increase fruits and vegetables. Exercise 30 minutes most days.

C D E Primary prevention of hypertension is to make lifestyle modifications that prevent or delay the increase in BP. Along with exercise for 30 minutes on most days, the DASH eating plan is a healthy way to lower BP by limiting sodium and fat intake, increasing fruits and vegetables, and increasing nutrients that are associated with lowering BP. Nuts and seeds and dried beans are used for protein intake. Weight loss may or may not be necessary for the individual.

1. Which instruction given to a patient who is about to undergo Holter monitoring is most appropriate? A "You may remove the monitor only to shower or bathe." B "You should connect the monitor whenever you feel symptoms." C "You should refrain from exercising while wearing this monitor." D "You will need to keep a diary of all your activities and symptoms."

D A Holter monitor is worn continuously for at least 24 hours while a patient continues with usual activity and keeps a diary of activities and symptoms. The patient should not take a bath or shower while wearing this monitor.

17. A patient experienced sudden cardiac death (SCD) and survived. What should the nurse expect to be used as preventive treatment for the patient? A External pacemaker B An electrophysiologic study (EPS) C Medications to prevent dysrhythmias D Implantable cardioverter-defibrillator (ICD)

D An ICD is the most common approach to preventing recurrence of SCD. An external pacemaker may be used in the hospital but will not be used for the patient living daily life at home. An EPS may be done to determine if a recurrence is likely and determine the most effective medication treatment. Medications to prevent dysrhythmias are used but are not the best prevention of SCD.

11. After having an MI, the nurse notes the patient has jugular venous distention, gained weight, developed peripheral edema, and has a heart rate of 108/minute. What should the nurse suspect is happening? A ADHF B Chronic HF C Left-sided HF D Right-sided HF

D An MI is a primary cause of heart failure. The jugular venous distention, weight gain, peripheral edema, and increased heart rate are manifestations of right-sided heart failure.

The blood pressure of a 71-year-old patient admitted with pneumonia is 160/70 mm Hg. What is an age-related change that contributes to this finding? A Stenosis of the heart valves B Decreased adrenergic sensitivity C Increased parasympathetic activity D Loss of elasticity in arterial vessels

D An age-related change that increases the risk of systolic hypertension is a loss of elasticity in the arterial walls. Because of the increasing resistance to flow, pressure is increased within the blood vessel, and hypertension results. Valvular rigidity of aging causes murmurs, and decreased adrenergic sensitivity slows the heart rate. Blood pressure is not raised. Increased parasympathetic activity would slow the heart rate.

3. A 44-year-old man is diagnosed with hypertension and receives a prescription for benazepril (Lotensin). After the nurse teaches him about the medication, which statement by the patient indicates his correct understanding? A "If I take this medication, I will not need to follow a special diet." B "It is normal to have some swelling in my face while taking this medication." C "I will need to eat foods such as bananas and potatoes that are high in potassium." D "If I develop a dry cough while taking this medication, I should notify my doctor."

D Benazepril is an angiotensin-converting enzyme inhibitor. The medication inhibits breakdown of bradykinin, which may cause a dry, hacking cough. Other adverse effects include hyperkalemia. Swelling in the face could indicate angioedema and should be reported immediately to the prescriber. Patients taking drug therapy for hypertension should also attempt lifestyle modifications to lower blood pressure such as a reduced-sodium diet.

12. Beyond the first year after a heart transplant, the nurse knows that what is a major cause of death? A Infection B Acute rejection C Immunosuppression D Cardiac vasculopathy

D Beyond the first year after a heart transplant, malignancy (especially lymphoma) and cardiac vasculopathy (accelerated CAD) are the major causes of death. During the first year after transplant, infection and acute rejection are the major causes of death. Immunosuppressive therapy will be used for posttransplant management to prevent rejection and increases the patient's risk of an infection.

11. What is a priority nursing intervention in the care of a patient with a diagnosis of chronic venous insufficiency (CVI)? A Application of topical antibiotics to venous ulcers B Maintaining the patient's legs in a dependent position C Administration of oral and/or subcutaneous anticoagulants D Teaching the patient the correct use of compression stockings

D CVI requires conscientious and consistent application of compression stockings. Anticoagulants are not necessarily indicated and antibiotics, if required, are typically oral or IV, not topical. The patient should avoid prolonged positioning with the limb in a dependent position.

8. When evaluating a patient's knowledge regarding a low-sodium, low-fat cardiac diet, the nurse recognizes additional teaching is needed when the patient selects which food choice? A Baked flounder B Angel food cake C Baked potato with margarine D Canned chicken noodle soup

D Canned soups are very high in sodium content. Patients need to be taught to read food labels for sodium and fat content.

2. A 74-year-old woman who is admitted with severe dyspnea has a history of heart failure and chronic obstructive lung disease. Which diagnostic study would the nurse expect to be elevated if the cause of dyspnea was cardiac related? A Serum potassium B Serum homocysteine C High-density lipoprotein D b-type natriuretic peptide (BNP)

D Elevation of b-type natriuretic peptide (BNP) indicates the presence of heart failure. Elevations help to distinguish cardiac vs. respiratory causes of dyspnea. Elevated potassium, homocysteine, or HDL levels may indicate increased risk for cardiovascular disorders but do not indicate that cardiac disease is present.

19. A male patient who has coronary artery disease (CAD) has serum lipid values of LDL cholesterol 98 mg/dL and HDL cholesterol 47 mg/dL. What should the nurse include in the patient teaching? A Consume a diet low in fats. B Reduce total caloric intake. C Increase intake of olive oil. D The lipid levels are normal.

D For men, the recommended LDL is less than 100 mg/dL, and the recommended level for HDL is greater than 40mg/dL. His normal lipid levels should be included in the patient teaching and encourage him to continue taking care of himself. Assessing his need for teaching related to diet should also be done.

In reviewing medication instructions with a patient being discharged on antihypertensive medications, which statement would be most appropriate for the nurse to make when discussing guanethidine (Ismelin)? A "A fast heart rate is a side effect to watch for while taking guanethidine." B "Stop the drug and notify your doctor if you experience any nausea or vomiting." C "Because this drug may affect the lungs in large doses, it may also help your breathing." D "Make position changes slowly, especially when rising from lying down to a standing position."

D Guanethidine is a peripheral-acting α-adrenergic antagonist and can cause marked orthostatic hypotension. For this reason, the patient should be instructed to rise slowly, especially when moving from a recumbent to a standing position. Support stockings may also be helpful. Tachycardia or lung effects are not evident with guanethidine.

18. A female patient who has type 1 diabetes mellitus has chronic stable angina that is controlled with rest. She states that over the past few months she has required increasing amounts of insulin. What goal should the nurse use to plan care that should help prevent cardiovascular disease progression? A Exercise almost every day. B Avoid saturated fat intake. C Limit calories to daily limit. D Keep Hgb A1C less than 7%.

D If the Hgb A1C is kept below 7%, this means that the patient has had good control of her blood glucose over the past 3 months. The patient indicates that increasing amounts of insulin are being required to control her blood glucose. This patient may not be adhering to the dietary guidelines or therapeutic regimen, so teaching about how to maintain diet, exercise, and medications to maintain stable blood glucose levels will be needed to achieve this goal.

16. The patient is being dismissed from the hospital after ACS and will be attending rehabilitation. What information does the patient need to be taught about the early recovery phase of rehabilitation? A Therapeutic lifestyle changes should become lifelong habits. B Physical activity is always started in the hospital and continued at home. C Attention will focus on management of chest pain, anxiety, dysrhythmias, and other complications. D Activity level is gradually increased under cardiac rehabilitation team supervision and with ECG monitoring.

D In the early recovery phase after the patient is dismissed from the hospital, the activity level is gradually increased under supervision and with ECG monitoring. The late recovery phase includes therapeutic lifestyle changes that become lifelong habits. In the first phase of recovery, activity is dependent on the severity of the angina or MI, and attention is focused on the management of chest pain, anxiety, dysrhythmias, and other complications. With early recovery phase, the cardiac rehabilitation team may suggest that physical activity be initiated at home, but this is not always done.

15. On return from surgery, the patient is wearing intermittent sequential compression stockings that he does not want to keep on. How should the nurse explain their necessity to the patient while he is on bed rest? A The socks keep the legs warm while the patient is not moving much. B The socks maintain the blood flow to the legs while the patient is on bed rest. C The socks keep the blood pressure down while the patient is stressed after surgery. D The socks provide compression of the veins to keep the blood moving back to the heart.

D Intermittent sequential compression stockings provide compression of the veins while the patient is not using skeletal muscles to compress the veins, which keeps the blood moving back to the heart and prevents blood pooling in the legs that could cause deep vein thrombosis. The warmth is not important. Blood flow to the legs is not maintained. Blood pressure is not decreased with the use of intermittent sequential compression stockings.

6. The patient with pericarditis is complaining of chest pain. After assessment, which intervention should the nurse expect to implement to provide pain relief? A Corticosteroids B Morphine sulfate C Proton pump inhibitor D Nonsteroidal antiinflammatory drugs

D Nonsteroidal antiinflammatory drugs (NSAIDs) will control pain and inflammation. Corticosteroids are reserved for patients already taking corticosteroids for autoimmune conditions or those who do not respond to NSAIDs. Morphine is not necessary. Proton pump inhibitors are used to decrease stomach acid to avoid the risk of GI bleeding from the NSAIDs.

12. The nurse knows the ventricular contractions are directly stimulated by which anatomic feature of the heart? A 1 B 2 C 3 D 4

D The Purkinje fibers move the electrical impulse or action potential through the walls of both ventricles triggering synchronized right and left ventricular contraction. The sinoatrial (SA) node initiates the electrical impulse that results in atrial contraction. The atrioventricular (AV) node receives the electrical impulse through internodal pathways. The bundle of His receives the impulse from the AV node.

5. A male patient with a long-standing history of heart failure has recently qualified for hospice care. What measure should the nurse now prioritize when providing care for this patient? A Taper the patient off his current medications. B Continue education for the patient and his family. C Pursue experimental therapies or surgical options. D Choose interventions to promote comfort and prevent suffering.

D The central focus of hospice care is the promotion of comfort and the prevention of suffering. Patient education should continue, but providing comfort is paramount. Medications should be continued unless they are not tolerated. Experimental therapies and surgeries are not commonly used in the care of hospice patients.

22. The patient reports tenderness when she touches her leg over a vein. The nurse assesses warmth and a palpable cord in the area. The nurse knows the patient needs treatment to prevent which sequelae? A Pulmonary embolism B Pulmonary hypertension C Post-thrombotic syndrome D Venous thromboembolism

D The clinical manifestations are characteristic of a superficial vein thrombosis. If untreated, the clot may extend to deeper veins, and venous thromboembolism may occur. Pulmonary embolism, pulmonary hypertension, and post-thrombotic syndrome are the sequelae of venous thromboembolism.

19. A male patient was admitted for a possible ruptured aortic aneurysm, but had no back pain. Ten minutes later his assessment includes the following: sinus tachycardia at 138, BP palpable at 65 mm Hg, increasing waist circumference, and no urine output. How should the nurse interpret this assessment about the patient's aneurysm? A Tamponade will soon occur. B The renal arteries are involved. C Perfusion to the legs is impaired. D He is bleeding into the abdomen.

D The lack of back pain indicates the patient is most likely exsanguinating into the abdominal space, and the bleeding is likely to continue without surgical repair. A blockade of the blood flow will not occur in the abdominal space as it would in the retroperitoneal space where surrounding anatomic structures may control the bleeding. The lack of urine output does not indicate renal artery involvement, but that the bleeding is occurring above the renal arteries, which decreases the blood flow to the kidneys. There is no assessment data indicating decreased perfusion to the legs.

9. A stable patient with acute decompensated heart failure (ADHF) suddenly becomes dyspneic. Before positioning the patient on the bedside, what should the nurse assess first? A Urine output B Heart rhythm C Breath sounds D Blood pressure

D The nurse should evaluate the blood pressure before dangling the patient on the bedside because the blood pressure can decrease as blood pools in the periphery and preload decreases. If the patient's blood pressure is low or marginal, the nurse should put the patient in the semi-Fowler's position and use other measures to improve gas exchange.

2. The nurse is preparing to administer a scheduled dose of enoxaparin (Lovenox) 30 mg subcutaneously. What should the nurse do to administer this medication correctly? A Remove the air bubble in the prefilled syringe. B Aspirate before injection to prevent IV administration. C Rub the injection site after administration to enhance absorption. D Pinch the skin between the thumb and forefinger before inserting the needle.

D The nurse should gather together or "bunch up" the skin between the thumb and the forefinger before inserting the needle into the subcutaneous tissue. The nurse should not remove the air bubble in the prefilled syringe, aspirate, nor rub the site after injection.

9. An 80-year-old patient with uncontrolled type 1 diabetes mellitus is diagnosed with aortic stenosis. When conservative therapy is no longer effective, the nurse knows that the patient will need to do or have what done? A Aortic valve replacement B Take nitroglycerin for chest pain. C Open commissurotomy (valvulotomy) procedure D Percutaneous transluminal balloon valvuloplasty (PTBV) procedure

D The percutaneous transluminal balloon valvuloplasty (PTBV) procedure is best for this older adult patient who is a poor surgery candidate related to the uncontrolled type 1 diabetes mellitus. Aortic valve replacement would probably not be tolerated well by this patient, although it may be done if the PTBV fails and the diabetes is controlled in the future. Nitroglycerin is used cautiously for chest pain because it can reduce BP and worsen chest pain in patients with aortic stenosis. Open commissurotomy procedure is used for mitral stenosis.

4. A 39-year-old woman with a history of smoking and oral contraceptive use is admitted with a venous thromboembolism (VTE) and prescribed unfractionated heparin. What laboratory test should the nurse review to evaluate the expected effect of the heparin? A Platelet count B Activated clotting time (ACT) C International normalized ratio (INR) D Activated partial thromboplastin time (APTT)

D Unfractionated heparin can be given by continuous IV for VTE treatment. When given IV, heparin requires frequent laboratory monitoring of clotting status as measured by activated partial thromboplastin time (aPTT). Platelet counts can decrease as an adverse reaction to heparin, but that is not the expected effect.

7. The nurse is caring for a patient with a recent history of deep vein thrombosis (DVT). The patient now needs to undergo surgery for appendicitis. The nurse is reviewing the laboratory results for this patient before administering an ordered dose of vitamin K. The nurse determines that the medication is both safe to give and is most needed when the international normalized ratio (INR) is which result? A 1.0 B 1.2 C 1.6 D 2.2

D Vitamin K is the antidote to warfarin (Coumadin), which the patient has most likely been taking before admission for treatment of DVT. Warfarin is an anticoagulant that impairs the ability of the blood to clot. Therefore it is necessary to give vitamin K before surgery to reduce the risk of hemorrhage. The largest value of the INR indicates the greatest impairment of clotting ability, making 2.2 the correct selection.

13. For which problem is percutaneous coronary intervention (PCI) most clearly indicated? A Chronic stable angina B Left-sided heart failure C Coronary artery disease D Acute myocardial infarction

D PCI is indicated to restore coronary perfusion in cases of myocardial infarction. Chronic stable angina and CAD are normally treated with more conservative measures initially. PCI is not relevant to the pathophysiology of heart failure, such as left-sided heart failure.

1. A 20-year-old patient has acute infective endocarditis. While obtaining a nursing history, what should the nurse ask the patient about (select all that apply)? a. Renal dialysis b. IV drug abuse c. Recent dental work d. Cardiac catheterization e. Recent urinary tract infection

a, b, c, d, e. Recent dental, urologic, surgical, or gynecologic procedures and history of IV drug abuse, heart disease, cardiac catheterization or surgery, renal dialysis, and infections all increase the risk of infective endocarditis.

9. What are characteristic of arteriospastic disease (Raynaud's phenomenon) (select all that apply)? a. Predominant in young females b. May be associated with autoimmune disorders c. Precipitated by exposure to cold, caffeine, and tobacco d. Involves small cutaneous arteries of the fingers and toes e. Inflammation of small and medium-sized arteries and veins f. Episodes involve white, blue, and red color changes of fingertips

a, b, c, d, f. Raynaud's phenomenon is predominant in young females and may be associated with autoimmune disorders (e.g., rheumatoid arthritis, scleroderma, systemic lupus erythematosus). Incidents occur with cold, emotional upsets, and caffeine or tobacco use due to vasoconstrictive effects. Small cutaneous arteries are involved and cause color changes of the fingertips or toes. When conservative management is ineffective, it may be treated with nifedipine (Procardia).

27. The surgery area calls the transfer report for a 68-year-old, postmenopausal, female patient who smokes and takes hormone therapy. She is returning to the floor after a lengthy hip replacement surgery. Which factors present in this patient increase her risk for developing venous thromboembolism (VTE) related to Virchow's triad (select all that apply)? a. Smoking b. IV therapy c. Dehydration d. Estrogen therapy e. Orthopedic surgery f. Prolonged immobilization

a, b, d, e, f. This patient is a smoker and on hormone therapy, both of which increase blood hypercoagulability. She will have an IV, which can cause VTE by damaging the venous endothelium. She is an older patient who has had an orthopedic surgery and may have experienced prolonged immobility postinjury and through her "lengthy hip replacement surgery," which contributes to venous stasis. These are representative of Virchow's triad in this patient. The other options are also related to Virchow's triad but not present in this patient via the transfer report.

1. Which statements accurately describe heart failure (select all that apply)? a. A common cause of diastolic failure is left ventricular hypertrophy. b. A primary risk factor for heart failure is coronary artery disease (CAD). c. Systolic heart failure results in a normal left ventricular ejection fraction. d. Systolic failure is characterized by abnormal resistance to ventricular filling. e. Hypervolemia precipitates heart failure by decreasing cardiac output and increasing oxygen consumption.

a, b. Diastolic failure is characterized by abnormal resistance to ventricular filling. Coronary artery disease (CAD), advanced age, and hypertension are all risk factors for heart failure (HF). Ejection fraction is decreased in systolic HF. Dysrthythmia precipitates HF with decreased cardiac output (CO) and increased workload and oxygen requirements of the myocardium.

4. What are nonmodifiable risk factors for primary hypertension (select all that apply)? a. Age d. Ethnicity b. Obesity e. Genetic link c. Gender

a, c, d, e. Hypertension progresses with increasing age. It is more prevalent in men up to age 45 and above the age of 64 in women. African Americans have a higher incidence of hypertension than do white Americans. Children and siblings of patients with hypertension should be screened and taught about healthy lifestyles.

16. A 78-year-old patient is admitted with a BP of 180/98 mm Hg. Which age-related physical changes may contribute to this patient's hypertension (select all that apply)? a. Decreased renal function b. Increased baroreceptor reflexes c. Increased peripheral vascular resistance d. Increased adrenergic receptor sensitivity e. Increased collagen and stiffness of the myocardium f. Loss of elasticity in large arteries from arteriosclerosis

a, c, e, f. The age-related changes that contribute to hypertension include decreased renal function, increased peripheral vascular resistance, increased collagen and stiffness of the myocardium, and decreased elasticity in large arteries from arteriosclerosis. The baroreceptor reflexes are blunted. The adrenergic receptor sensitivity and renin response are both decreased with aging.

9. During a routine health examination, a 48-year-old patient is found to have a total cholesterol level of 224 mg/dL (5.8 mmol/L) and an LDL level of 140 mg/dL (3.6 mmol/L). What does the nurse teach the patient based on the Therapeutic Lifestyle Changes diet (select all that apply)? a. Use fat-free milk d. Eliminate intake of simple sugars b. Abstain from alcohol use e. Avoid egg yolks and foods prepared with whole eggs c. Reduce red meat in the diet

a, c, e. Therapeutic Lifestyle Changes diet recommendations emphasize reduction in saturated fat and cholesterol intake. Red meats, whole milk products, and eggs as well as butter, stick margarine, lard, and solid shortening should be reduced or eliminated from diets. If triglyceride levels are high, alcohol and simple sugars should be reduced.

4. A patient with PAD has a nursing diagnosis of ineffective peripheral tissue perfusion. What should be included in the teaching plan for this patient (select all that apply)? a. Keep legs and feet warm. b. Apply cold compresses when the legs become swollen. c. Walk at least 30 minutes per day to the point of discomfort. d. Use nicotine replacement therapy as a substitute for smoking. e. Inspect lower extremities for pulses, temperature, and any injury.

a, c, e. Warm legs and feet increase circulation. The lower extremities should be assessed at regular intervals for changes. Walking exercise increases oxygen extraction in the legs and improves skeletal muscle metabolism. The patient with PAD should walk at least 30 minutes a day, preferably twice a day. Exercise should be stopped when pain occurs and resumed when the pain subsides. Nicotine in all forms causes vasoconstriction and must be eliminated.

32. Which characteristics describe the anticoagulant warfarin (Coumadin) (select all that apply)? a. Vitamin K is the antidote b. Protamine sulfate is the antidote c. May be administered orally or subcutaneously d. May be administered intravenously or subcutaneously e. Dosage monitored using international normalized ratio (INR) f. Dosage monitored using activated partial thromboplastin time (aPTT)

a, e. Warfarin (Coumadin) is a vitamin K antagonist, so vitamin K is the antidote. It is monitored with the international normalized ratio (INR). It is only administered orally. Protamine sulfate is the antidote for unfractionated heparin (Heparin), which can be administered subcutaneously or IV and is monitored with activated partial thromboplastin time (aPTT). Hirudin derivatives are given IV or subcutaneously, do not have an antidote, and are also monitored with aPTT. Argatroban (Acova), a synthetic direct thrombin inhibitor, is given only IV and is monitored with aPTT. Factor Xa inhibitor fondaparinux (Arixtra) is given subcutaneously and does not require routine coagulation testing. Rivaroxaban (Xarelto), another factor Xa inhibitor, is given orally.

36. Number in sequence the processes that occur as venous stasis leads to varicose veins and to venous stasis ulcers.

a. 2; b. 7; c. 9; d. 1; e. 6; f. 4; g. 5; h. 3; i. 8

20. When instructing the patient with angina about taking sublingual nitroglycerin tablets, what should the nurse teach the patient? a. To lie or sit and place one tablet under the tongue when chest pain occurs b. To take the tablet with a large amount of water so it will dissolve right away c. That if one tablet does not relieve the pain in 15 minutes, the patient should go to the hospital d. That if the tablet causes dizziness and a headache, stop the medication and call the doctor or go to the hospital

a. A common complication of nitrates is dizziness caused by orthostatic hypotension, so the patient should sit or lie down and place the tablet under the tongue. The tablet should be allowed to dissolve under the tongue. To prevent the tablet from being swallowed, water should not be taken with it. The recommended dose for the patient for whom nitroglycerin (NTG) has been prescribed is one tablet taken sublingually (SL) or one metered spray for symptoms of angina. If symptoms are unchanged or worse after 5 minutes, the patient should contact the emergency medical services (EMS) system before taking additional NTG. If symptoms are significantly improved by one dose of NTG, instruct the patient or caregiver to repeat NTG every 5 minutes for a maximum of three doses and contact EMS if symptoms have not resolved completely. Headache is also a common complication of nitrates but usually resolves with continued use of nitrates and may be controlled with mild analgesics.

23. A patient with a dissection of the arch of the aorta has a decreased LOC and weak carotid pulses. What should the nurse anticipate that initial treatment of the patient will include? a. Immediate surgery to replace the torn area with a graft b. Administration of anticoagulants to prevent embolization c. Administration of packed red blood cells (RBCs) to replace blood loss d. Administration of antihypertensives to maintain a mean arterial pressure of 70 to 80 mm Hg

a. Although most initial treatment for aortic dissection involves a period of lowering the BP and myocardial contractility to diminish the pulsatile forces in the aorta, immediate surgery is indicated when complications (such as occlusion of the carotid arteries) occur. Anticoagulants would prolong and intensify the bleeding and blood is administered only if the dissection ruptures.

45. A 58-year-old patient is in a cardiac rehabilitation program. The nurse teaches the patient to stop exercising if what occurs? a. Pain or dyspnea develop c. The respiratory rate increases to 30 b. The HR exceeds 150 bpm d. The HR is 30 bpm over the resting HR

a. Any activity or exercise that causes dyspnea and chest pain should be stopped in the patient with CAD. The training target for a healthy 58-year-old is 80% of maximum HR, or 130 bpm. In a patient with cardiac disease undergoing cardiac conditioning, however, the HR should not exceed 20 bpm over the resting pulse rate. HR, rather than respiratory rate, determines the parameters for exercise.

23. The nurse plans long-term goals for the patient who has had a heart transplant with the knowledge that what is the most common cause of death in heart transplant patients during the first year? a. Infection c. Embolization b. Heart failure d. Malignant conditions

a. Because of the need for long-term immunosuppressant therapy to prevent rejection, the patient with a transplant is at high risk for infection, a leading cause of death in transplant patients. Acute rejection episodes may also cause death in patients with transplants but many can be treated successfully with augmented immunosuppressive therapy. Malignancies occur in patients with organ transplants after taking immunosuppressants for a number of years.

2. A patient has an admitting diagnosis of acute left-sided infective endocarditis. What is the best test to confirm this diagnosis? a. Blood cultures b. Complete blood count c. Cardiac catheterization d. Transesophageal echocardiogram

a. Blood cultures are the primary diagnostic tool for infective endocarditis. Although a complete blood count (CBC) will reveal a mild leukocytosis, this is a nonspecific finding. Transesophageal echocardiograms can identify vegetations on valves but are used when blood cultures are negative. Cardiac catheterizations are used when surgical intervention is being considered.

17. What should the nurse emphasize when teaching a patient who is newly prescribed clonidine (Catapres)? a. The drug should never be stopped abruptly. b. The drug should be taken early in the day to prevent nocturia. c. The first dose should be taken when the patient is in bed for the night. d. Because aspirin will decrease the drug's effectiveness, Tylenol should be used instead.

a. Centrally acting α-adrenergic blockers may cause severe rebound hypertension if the drugs are abruptly discontinued and patients should be taught about this effect because many are not consistently compliant with drug therapy. Diuretics should be taken early in the day to prevent nocturia and the profound orthostatic hypotension that occurs with first-dose α-adrenergic blockers can be prevented by taking the initial dose at bedtime. Aspirin use may decrease the effectiveness of ACE inhibitors.

6. Which initial physical assessment finding would the nurse expect to be present in a patient with acute left-sided heart failure? a. Bubbling crackles and tachycardia b. Hepatosplenomegaly and tachypnea c. Peripheral edema and cool, diaphoretic skin d. Frothy blood-tinged sputum and distended jugular veins

a. Clinical manifestations of acute left-sided heart failure are those of interstitial edema, with bubbling crackles and tachycardia, as well as tachypnea. Later frothy, blood- tinged sputum; severe dyspnea; and orthopnea develop with alveolar edema. Severe tachycardia and cool, clammy skin are present as a result of stimulation of the sympathetic nervous system from hypoxemia. Systemic edema reflected by jugular vein distention, peripheral edema, and hepatosplenomegaly are characteristic of right-sided heart failure.

44. The nurse and patient set a patient outcome that at the time of discharge after an MI the patient will be able to tolerate moderate-energy activities that are similar to which activity? a. Golfing c. Cycling at 13 mph b. Walking at 5 mph d. Mowing the lawn by hand

a. Golfing is a moderate-energy activity that expends about 5 metabolic equivalent units (METs) and is within the 3 to 5 METs activity level desired for a patient by the time of discharge from the hospital following an MI. Walking at 5 mph and mowing the lawn by hand are high-energy activities and cycling at 13 mph is an extremely high-energy activity.

6. What is the patient with primary hypertension likely to report? a. No symptoms b. Cardiac palpitations c. Dyspnea on exertion d. Dizziness and vertigo

a. Hypertension is often asymptomatic, especially if it is mild or moderate, and has been called the "silent killer." The absence of symptoms often leads to noncompliance with medical treatment and a lack of concern about the disease in patients. With severe hypertension, symptoms usually occur and may include a morning occipital headache, fatigability, dizziness, palpitations, angina, and dyspnea.

37. The nurse recognizes that thrombolytic therapy for the treatment of an MI has not been successful when the patient displays which manifestation? a. Continues to have chest pain b. Has a marked increase in CK enzyme levels within 3 hours of therapy c. Develops major gastrointestinal (GI) or genitourinary (GU) bleeding during treatment d. Develops premature ventricular contractions and ventricular tachycardia during treatment

a. If chest pain is unchanged, it is an indication that reperfusion was not successful. Indications that the occluded coronary artery is patent and blood flow to the myocardium is reestablished following thrombolytic therapy include return of ST segment to baseline on the ECG; relief of chest pain; marked, rapid rise of the CK enzyme within 3 hours of therapy; and the presence of reperfusion dysrhythmias.

18. A patient is scheduled for exercise nuclear imaging stress testing. The nurse explains to the patient that this test involves a. IV administration of a radioisotope at the maximum heart rate during exercise to identify the heart's response to physical stress. b. placement of electrodes inside the right-sided heart chambers through a vein to record the electrical activity of the heart directly. c. exercising on a treadmill or stationary bicycle with continuous ECG monitoring to detect ischemic changes during exercise. d. placement of a small transducer in four positions on the chest to record the direction and flow of blood through the heart by the reflection of sound waves.

a. In an exercise nuclear imaging scan, a radioisotope is injected at the maximum heart rate on a bicycle or treadmill and used to evaluate blood flow in different parts of the heart. Insertion of electrodes into the heart chambers via the venous system to record intracardiac electrical activity is an electrophysiology study. Simply monitoring ECG activity during exercise is an exercise stress test and an echocardiogram uses transducers to bounce sound waves off of the heart.

8. The nurse is encouraging a sedentary patient with major risks for CAD to perform physical exercise on a regular basis. In addition to decreasing the risk factor of physical inactivity, the nurse tells the patient that exercise will also directly contribute to reducing which risk factors? a. Hyperlipidemia and obesity c. Elevated serum lipids and stressful lifestyle b. Diabetes mellitus and hypertension d. Hypertension and elevated serum homocysteine

a. Increased exercise without an increase in caloric intake will result in weight loss, reducing the risk associated with obesity. Exercise increases lipid metabolism and increases HDL, thus reducing CAD risk. Exercise may also indirectly reduce the risk of CAD by controlling hypertension, promoting glucose metabolism in diabetes, and reducing stress. Although research is needed to determine whether a decline in homocysteine can reduce the risk of heart disease, it appears that dietary modifications are indicated for risk reduction.

22. During treatment of a patient with a BP of 222/148 mm Hg and confusion, nausea, and vomiting, the nurse initially titrates the medications to achieve which goal? a. Decrease the mean arterial pressure (MAP) to 129 mm Hg b. Lower the BP to the patient's normal within the second to third hour d. Reduce the systolic BP (SBP) to 158 mm Hg and the diastolic BP (DBP) to 111 mm Hg within the first 2 hours c. Decrease the SBP to 160 mm Hg and the DBP to between 100 and 110 mm Hg as quickly as possible

a. Initially the treatment goal in hypertensive emergencies is to reduce the mean arterial pressure (MAP) by no more than 20% to 25% in the first hour, with further gradual reduction over the next 24 hours. In this case the MAP is 172, so decreasing it by 25% equals 129. Lowering the BP too far or too fast may cause a stroke, myocardial infarction (MI), visual changes, or renal failure. Only when the patient has an aortic dissection, angina, or signs of MI or an ischemic stroke does the SBP need to be lowered to 100 to 120 mm Hg or less as quickly as possible.

16. What manifestations most strongly support a diagnosis of acute rheumatic fever? a. Carditis, polyarthritis, and erythema marginatum b. Polyarthritis, chorea, and decreased antistreptolysin O titer c. Organic heart murmurs, fever, and elevated erythrocyte sedimentation rate (ESR) d. Positive C-reactive protein, elevated white blood cells (WBCs), and subcutaneous nodules

a. Major criteria for the diagnosis of rheumatic fever include evidence of carditis, polyarthritis, chorea (often very late), erythema marginatum, and subcutaneous nodules. Minor criteria include all laboratory findings as well as fever, arthralgia, and a history of previous rheumatic fever. There also must be evidence of a previous group A streptococci infection (e.g., positive antistreptolysin O titer).

48. Priority Decision: A patient is hospitalized after a successful resuscitation of an episode of sudden cardiac death (SCD). During the care of the patient, what nursing intervention is most important? a. Continuous ECG monitoring c. Frequent assessment of heart sounds b. Auscultation of the carotid arteries d. Monitoring of airway status and respiratory patterns

a. Most patients who experience sudden cardiac death (SCD) as a result of CAD do not have an acute MI but have dysrhythmias that cause death, probably as a result of electrical instability of the myocardium. To identify and treat those specific dysrhythmias, continuous monitoring is important. The other assessments can be done but are not the most important after an episode of SCD.

23. The nurse suspects stable angina rather than MI pain in the patient who reports that his chest pain a. is relieved by nitroglycerin. c. does not radiate to the neck, back, or arms. b. is a sensation of tightness or squeezing. d. is precipitated by physical or emotional exertion.

a. One of the primary differences between the pain of angina and the pain of an MI is that angina pain is usually relieved by rest or nitroglycerin, which reduces the oxygen demand of the heart, whereas MI pain is not. Both angina and MI pain can cause a pressure or squeezing sensation; may or may not radiate to the neck, back, arms, fingers, and jaw; and may be precipitated by exertion.

21. When teaching an older adult with CAD how to manage the treatment program for angina, which guidelines does the nurse use to teach the patient? a. To sit for 2 to 5 minutes before standing when getting out of bed b. To exercise only twice a week to avoid unnecessary strain on the heart c. That lifestyle changes are not as necessary as they would be in a younger person d. That aspirin therapy is contraindicated in older adults because of the risk for bleeding

a. Orthostatic hypotension may cause dizziness and falls in older adults taking antianginal agents that decrease preload. Patients should be cautioned to change positions slowly. Daily exercise programs are indicated for older adults and may increase performance, endurance, and ability to tolerate stress. A change in lifestyle behaviors may increase the quality of life and reduce the risks of CAD, even in the older adult. Aspirin is commonly used in these patients and is not contraindicated.

7. Priority Decision: A patient has atrial fibrillation and develops an acute arterial occlusion at the iliac artery bifurcation. What are the six Ps of acute arterial occlusion the nurse may assess in this patient that require immediate notification of the physician? a. b. c. d. e. f.

a. Pain b. pallo c. pulselessness d. paresthesia e. paralysis f. poikilothermia The physician requires immediate notification to begin immediate intervention to prevent tissue necrosis and gangrene.

3. Which manifestation of infective endocarditis is a result of fragmentation and microembolization of vegetative lesions? a. Petechiae b. Roth's spots c. Osler's nodes d. Splinter hemorrhages

a. Petechiae are seen as small hemorrhages in the conjunctiva, lips, and buccal mucosa and over the ankles, feet, and antecubital and popliteal areas. Roth's spots are hemorrhagic retinal lesions seen with funduscopic examination. Osler's nodes are lesions on the fingertips or toes. The cause of Roth's spots and Osler's nodes is not clear. Splinter hemorrhages are black longitudinal streaks that occur on nail beds. They may be caused by vessel damage from vasculitis or microemboli.

24. The nurse evaluates that treatment for the patient with an uncomplicated aortic dissection is successful when what happens? a. Pain is relieved. b. Surgical repair is completed. c. BP is increased to normal range. d. Renal output is maintained at 30 mL/hr.

a. Relief of pain is an indication that the dissection has stabilized and it may be treated conservatively for an extended time with drugs that lower the BP and decrease myocardial contractility. Surgery is usually indicated for dissection of the ascending aorta or if complications occur.

8. Priority Decision: The nurse reviews the following vital signs recorded by an unlicensed assistive personnel (UAP) on a patient with acute decompensated heart failure: BP 98/60, HR 102 bpm, RR 24, Temp 98.2°F (36.7° C), SpO2 84% on 2 L/min via nasal cannula. a. Which of these findings is of highest priority? b. What should the nurse do next?

a. SpO2 of 84% on 2 L/min via nasal cannula indicates impaired oxygen saturation. The patient is having trouble with gas exchange. Airway and breathing are the priority (follow ABCs). b. The nurse should place the patient in high Fowler's position, assess the patient immediately, recheck SpO2, auscultate breath sounds, assess level of consciousness (LOC), check the oxygen connection and rate setting (2 L/min), and talk with the patient about her or his breathing.

16. The health care provider prescribes spironolactone (Aldactone) for the patient with chronic heart failure. What diet modifications related to the use of this drug should the nurse include in the patient teaching? a. Decrease both sodium and potassium intake b. Increase calcium intake and decrease sodium intake c. Decrease sodium intake and increase potassium intake d. Decrease sodium intake and the use of salt substitutes for seasoning

a. Spironolactone is a potassium-sparing diuretic and when it is the only diuretic used in the treatment of heart failure, moderate to low levels of potassium intake should be maintained to prevent development of hyperkalemia. Sodium intake is usually reduced to at least 2400 mg/day in patients with heart failure but salt substitutes cannot be freely used because most contain high concentrations of potassium. Calcium intake is not increased.

10. To which patients should the nurse teach the Therapeutic Lifestyle Changes diet to reduce the risk of coronary artery disease (CAD)? a. All patients to reduce CAD risk b. Patients who have experienced an MI c. Individuals with two or more risk factors for CAD d. Individuals with a cholesterol level >200 mg/dL (5.2 mmol/L)

a. The Therapeutic Lifestyle Changes diet includes recommendations for all people, not just those with risk factors, to decrease the risk for CAD.

41. A patient with an MI is exhibiting anxiety while being taught about possible lifestyle changes. The nurse evaluates that the anxiety is relieved when the patient states a. "I'm going to take this recovery one step at a time." b. "I feel much better and am ready to get on with my life." c. "How soon do you think I will be able to go back to work?" d. "I know you are doing everything possible to save my life."

a. This patient is indicating positive coping with a realization that recovery takes time and that lifestyle changes can be made as needed. The patient who is "just going to get on with life" is probably in denial about the seriousness of the condition and the changes that need to be made. Nervous questioning about the expected duration and effect of the condition indicates the presence of anxiety, as does the statement regarding the health care professional's role in treatment.

16. In preparation for AAA repair surgery, what should the nurse include in patient teaching? a. Prepare the bowel on the night before surgery with laxatives or an enema. b. Use moisturizing soap to clean the skin three times the day before surgery. c. Eat a high-protein and high-carbohydrate breakfast to help with healing postoperatively. d. Take the prescribed oral antibiotic the morning of surgery before going to the operating room.

a. Usually aortic surgery patients will have a bowel preparation, skin cleansing with an antimicrobial agent on the day before surgery, nothing by mouth after midnight on the day of the surgery, and IV antibiotics immediately before the incision is made. Patients with a history of cardiovascular disease will receive a β-adrenergic blocker preoperatively to reduce morbidity and mortality. Each surgeon's protocol may be different.

28. The patient is admitted with pain, edema, and warm skin on her lower left leg. What test should the nurse expect to be ordered first? a. Duplex ultrasound b. Complete blood count (CBC) c. Magnetic resonance imaging (MRI) d. Computed venography (phlebogram)

a. With manifestations of a VTE, the D-dimer is drawn to determine if a VTE exists and the duplex ultrasound is most widely used to diagnose VTE by identifying where a thrombus is located and its extent.

15. Which surgical therapy for AAA is most likely to have the postoperative complication of renal injury? a. Open aneurysm repair (OAR) above the level of the renal arteries b. Excising only the weakened area of the artery and suturing the artery closed c. Bifurcated graft used in aneurysm repair when the AAA extends into the iliac arteries d. Endovascular graft procedure with an aortic graft inside the aneurysm via the femoral artery

a. With the aortic cross-clamping proximal and distal to the aneurysm, the open aneurysm repair (OAR) above the renal artery may cause kidney injury from lack of blood flow during the surgery. The saccular aneurysm may involve excising only the weakened area of the artery and suturing the artery closed but this will not decrease renal blood flow. Renal blood flow will not be directly obstructed using the bifurcated graft or the minimally invasive endovascular aneurysm repair.

18. Which effects contribute to making nitrates the first-line therapy for the treatment of angina (select all that apply)? a. Decrease heart rate (HR) b. Decrease preload c. Decrease afterload d. Dilate coronary arteries e. Prevent thrombosis of plaques f. Decrease myocardial contractility

b, c, d. Nitrates decrease preload and afterload to decrease the coronary workload and dilate coronary arteries to increase coronary blood supply. The other options are not attributed to nitrates.

5. Which characteristics are associated with LDLs (select all that apply)? a. Increases with exercise b. Contains the most cholesterol c. Has an affinity for arterial walls d. Carries lipids away from arteries to liver e. High levels correlate most closely with CAD f. The higher the level, the lower the risk for CAD

b, c, e. LDLs contain more cholesterol than the other lipoproteins, have an attraction for arterial walls, and correlate most closely with increased incidence of atherosclerosis and CAD. HDLs increase with exercise and carry lipids away from arteries to the liver for metabolism. A high HDL level is associated with a lower risk of CAD.

16. Which characteristics describe unstable angina (select all that apply)? a. Usually precipitated by exertion d. Occurs only when the person is recumbent b. Unpredictable and unrelieved by rest e. Usually occurs in response to coronary artery spasm c. Characterized by progressive severity

b, c. Unstable angina is unpredictable and unrelieved by rest and has progressively increasing severity. Chronic stable angina is usually precipitated by exertion. Angina decubitus occurs when the person is recumbent. Prinzmetal's angina is frequently caused by a coronary artery spasm.

25. What are characteristics of arterial disease (select all that apply)? a. Pruritus b. Thickened, brittle nails c. Dull ache in calf or thigh d. Decreased peripheral pulses e. Pallor on elevation of the legs f. Ulcers over bony prominences on toes and feet

b, d, e, f. Arterial disease is manifested in thick, brittle nails; decreased peripheral pulses; pallor when the legs are elevated; and ulcers over bony prominences on the toes and feet, as well as paresthesia. The other options are characteristic of venous disease and paresthesia could occur with venous thromboembolism (VTE).

13. Myocardial ischemia occurs as a result of increased oxygen demand and decreased oxygen supply. What factors and disorders result in increased oxygen demand (select all that apply)? a. Hypovolemia or anemia b. Increased cardiac workload with aortic stenosis c. Narrowed coronary arteries from atherosclerosis d. Angina in the patient with atherosclerotic coronary arteries e. Left ventricular hypertrophy caused by chronic hypertension f. Sympathetic nervous system stimulation by drugs, emotions, or exertion

b, d, e, f. Increased oxygen demand is caused by increasing the workload of the heart, including left ventricular hypertrophy with hypertension, sympathetic nervous stimulation, and anything precipitating angina. Hypovolemia, anemia, and narrowed coronary arteries contribute to decreased oxygen supply.

23. Increases in which factors are predictors of an increased risk for coronary artery disease or evidence of myocardial injury (select all that apply)? a. Creatine kinase (CK)-MM b. Cardiac troponin T (cTnT) c. B-type natriuretic peptide (BNP) d. High-sensitivity C-reactive protein e. Lipoprotein-associated phospholipase A2

b, d, e. Increased levels of cardiac tropinin T (cTnT), high-sensitivity C-reactive protein, and lipoprotein- associated phospholipase A2 are indicators of risk for or evidence of myocardial injury. Increased b-type natriuretic peptide (BNP) is a marker for heart failure and increased CK-creatine kinase (CK)-MM is most commonly associated with skeletal muscle injury.

8. Which conditions characterize critical limb ischemia (select all that apply)? a. Cold feet b. Arterial leg ulcers c. Venous leg ulcers d. Gangrene of the leg e. No palpable peripheral pulses f. Rest pain lasting more than 2 weeks

b, d, f. Arterial leg ulcers and/or gangrene of the leg due to PAD and chronic ischemic rest pain lasting more than 2 weeks characterize critical limb ischemia. Optimal therapy is revascularization via bypass surgery.

14. When caring for the patient with heart failure, which medications or treatments require careful monitoring of the patient's serum potassium level to prevent further cardiac dysfunction (select all that apply)? a. Enalapril (Vasotec) d. Spironolactone (Aldactone) b. Furosemide (Lasix) e. Metoprolol CR/XL (Toprol XL)

b, d. Furosemide is a diuretic that eliminates potassium and spironalactone is a potassium-sparing diuretic that retains potassium. The other treatments and medications are used for patients with HF but they do not directly affect serum potassium levels.

11. What explains the measurement of pulsus paradoxus with cardiac tamponade (select all that apply)? a. A difference of less than 10 mm Hg occurs. b. A difference of greater than 10 mm Hg occurs. c. It is measured with an automatic sphygmomanometer. d. Rapidly inflate the cuff until you hear sounds throughout the respiratory cycle. e. Subtract the number when sounds are heard in the respiratory cycle from the number when the first Korotkoff sound during expiration is heard.

b, e. Pulsus paradoxus is measured with a manually operated sphygmomanometer. The cuff is deflated slowly until the first Korotkoff sound during expiration is heard and the number is noted. The slow deflation of the cuff is continued until sounds are heard throughout the respiratory cycle and that number is subtracted from the first number. When the difference is >10 mm Hg, cardiac tamponade may be present. The difference is normally <10 mm Hg.

37. What is the most important measure in the treatment of venous stasis ulcers? a. Elevation of the limb b. Elastic compression stockings c. Application of moist dressings d. Application of topical antibiotics

b. Although leg elevation, moist dressings, and topical antibiotics are useful in treatment of venous stasis ulcers, the most important factor appears to be elastic compression stockings, which provide extrinsic compression to minimize venous stasis, venous hypertension, and edema.

24. A patient admitted to the hospital for evaluation of chest pain has no abnormal serum cardiac markers 4 hours after the onset of pain. What noninvasive diagnostic test can be used to differentiate angina from other types of chest pain? a. 12-lead ECG c. Coronary angiogram b. Exercise stress test d. Transesophageal echocardiogram

b. An exercise stress test will reveal ECG changes that indicate impaired coronary circulation when the oxygen demand of the heart is increased. A single ECG is not conclusive for CAD and negative findings do not rule out CAD. Coronary angiography can detect narrowing of coronary arteries but is an invasive procedure. Echocardiograms of various types may identify abnormalities of myocardial wall motion under stress but are indirect measures of CAD.

33. The patient with VTE is receiving therapy with heparin and asks the nurse whether the drug will dissolve the clot in her leg. What is the best response by the nurse? a. "This drug will break up and dissolve the clot so that circulation in the vein can be restored." b. "The purpose of the heparin is to prevent growth of the clot or formation of new clots where the circulation is slowed." c. "Heparin won't dissolve the clot but it will inhibit the inflammation around the clot and delay the development of new clots." d. "The heparin will dilate the vein, preventing turbulence of blood flow around the clot that may cause it to break off and travel to the lungs."

b. Anticoagulant therapy with heparin or warfarin (Coumadin) does not dissolve clots but prevents propagation of the clot, development of new thrombi, and embolization. Clot lysis occurs naturally through the body's intrinsic fibrinolytic system or by the administration of fibrinolytic agents.

23. The patient is admitted with angina, syncope, and dyspnea on exertion. In the assessment, the nurse notes a systolic murmur with a prominent S4. What will the nurse suspect is occurring with this patient? a. Mitral valve stenosis b. Aortic valve stenosis c. Acute mitral valve regurgitation d. Chronic mitral valve regurgitation

b. Aortic valve stenosis is identified with the triad of angina, syncope, and dyspnea on exertion, as well as the systolic murmur and prominent S4 heart sound. Mitral valve stenosis manifests as exertional dyspnea, hemoptysis, fatigue, atrial fibrillation, and a diastolic murmur. Acute mitral valve regurgitation has a new systolic murmur with pulmonary edema and cardiogenic shock rapidly developing. Chronic mitral valve regurgitation is identified with weakness, fatigue, exertional dyspnea, palpitations, an S3 gallop, and holosystolic murmur.

14. During preoperative preparation of the patient scheduled for an AAA, why should the nurse establish baseline data for the patient? a. All physiologic processes will be altered postoperatively. b. The cause of the aneurysm is a systemic vascular disease. c. Surgery will be canceled if any physiologic function is not normal. d. BP and HR will be maintained well below baseline levels during the postoperative period.

b. Because atherosclerosis is a systemic disease, the patient with an A A A is likely to have cardiac, pulmonary, cerebral, or lower extremity vascular problems that should be noted and monitored throughout the perioperative period. Postoperatively, the BP is balanced: high enough to keep adequate flow through the artery to prevent thrombosis but low enough to prevent bleeding at the surgical site.

19. The patient has used sublingual nitroglycerin and various long-acting nitrates but now has an ejection fraction of 38% and is considered at a high risk for a cardiac event. Which medication would first be added for vasodilation and to reduce ventricular remodeling? a. Clopidogrel (Plavix) c. Diltiazem (Cardizem) b. Captopril (Capoten) d. Metoprolol (Lopressor)

b. Captopril (Capoten) would be added. It is an angiotensin- converting enzyme (ACE) inhibitor that vasodilator and decreases endothelial dysfunction and may prevent ventricular remodeling. Clopidogrel (Plavix) is an antiplatelet agent used as an alternative for a patient unable to use aspirin. Diltiazem (Cardizem), a calcium channel blocker, may be used to decrease vasospasm but is not known to prevent ventricular remodeling. Metoprolol (Lopressor) is a β-adrenergic blocker that inhibits sympathetic nervous stimulation of the heart.

25. Priority Decision: A patient with symptomatic mitral valve prolapse has atrial and ventricular dysrhythmias. In addition to monitoring for decreased cardiac output related to the dysrhythmias, what is an important nursing intervention related to the dysrhythmias identified by the nurse? a. Monitor breathing pattern related to hypervolemia. b. Encourage calling for assistance when getting out of bed. c. Give sleeping pills to decrease paroxysmal nocturnal dyspnea. d. Teach the patient exercises to prevent recurrence of dysrhythmias.

b. Dysrhythmias frequently cause palpitations, lightheadedness, and dizziness and the patient should be carefully attended to to prevent falls. Hypervolemia and paroxysmal nocturnal dyspnea (PND) would be apparent in the patient with heart failure. Exercises will not prevent dysrhythmias.

5. A patient with infective endocarditis of a prosthetic mitral valve develops a left hemiparesis and visual changes. What should the nurse expect to be included in collaborative management of the patient? a. Embolectomy b. Surgical valve replacement c. Administration of anticoagulants d. Higher than usual antibiotic dosages

b. Early valve replacement followed by prolonged antibiotic and anticoagulant therapy is recommended for these patients. Drug therapy for patients who develop endocarditis of prosthetic valves is often unsuccessful in eliminating the infection and preventing embolization.

7. What is most organ damage in hypertension related to? a. Increased fluid pressure exerted against organ tissue b. Atherosclerotic changes in vessels that supply the organs c. Erosion and thinning of blood vessels from constant pressure d. Increased hydrostatic pressure causing leakage of plasma into organ interstitial spaces

b. Elevated BP causes the entire inner lining of arterioles to become thickened from hyperplasia of connective tissues in the intima and affects coronary circulation, cerebral circulation, peripheral vessels, and renal and retinal blood vessels. The narrowed vessels lead to ischemia and ultimately to damage of these organs.

6. Which serum lipid elevation, along with elevated LDL, is strongly associated with CAD? a. Apolipoproteins b. Fasting triglycerides c. Total serum cholesterol d. High-density lipoprotein (HDL)

b. Elevated fasting triglyceride levels are associated with cardiovascular disease and diabetes. Apolipoproteins are found in varying amounts on the HDLs and activate enzyme or receptor sites that promote removal of fat from plasma, which is protective. The apolipoprotein A and apolipropotein B ratio must be done to predict CAD. Elevated HDLs are associated with a lower risk of CAD. Elevated total serum cholesterol must be calculated with HDL for a ratio over time to determine an increased risk of CAD.

28. During the assessment, the nurse identifies crackles in the lungs and an S3 heart sound. Which complication of MI should the nurse suspect and further investigate? a. Pericarditis c. Ventricular aneurysm b. Heart failure d. Papillary muscle dysfunction

b. Heart failure, which can escalate to cardiogenic shock, initially occurs with mild dyspnea, restlessness, agitation, pulmonary congestion with crackles, S3 or S4 heart sounds, and jugular vein distention. Pericarditis is a common complication identified with chest pain that is aggravated by inspiration, coughing, and moving the upper body. Ventricular aneurysm is manifested with heart failure, dysrhythmias, and angina. Papillary muscle dysfunction is suspected with a new systolic apical murmur.

20. Which method is used to evaluate the ECG responses to normal activity over a period of 1 or 2 days? a. Serial ECGs b. Holter monitoring c. 6-minute walk test d. Event monitor or loop recorder

b. Holter monitoring involves placing electrodes on the chest attached to a recorder that will record ECG rhythm for 24 to 48 hours while the patient engages in normal activities of daily living (ADLs). The recording is later analyzed for cardiac dysrhythmias. Serial ECGs are frequent but not continuous ECGs. The 6-minute walk test measures the distance walked in 6 minutes to determine response to treatments and functional capacity for ADLs. An event monitor or loop recorder is used to record infrequent rhythm disturbances when the patient activates the recording with symptom occurrence.

15. Priority Decision: A patient with chronic heart failure is treated with hydrochlorothiazide, digoxin, and lisinopril (Prinivil). To prevent the risk of digitalis toxicity with these drugs, what is most important that the nurse monitor for this patient? a. Heart rate (HR) c. Blood pressure (BP) b. Potassium levels d. Gastrointestinal function

b. Hypokalemia is one of the most common causes of digitalis toxicity because low serum potassium levels enhance ectopic pacemaker activity. When a patient is receiving potassium-losing diuretics, such as hydrochlorothiazide or furosemide, it is essential to monitor the patient's serum potassium levels to prevent digitalis toxicity. Monitoring the heart rate (HR) assesses for complications related to digoxin but does not prevent toxicity.

26. The patient is diagnosed with a superficial vein thrombosis (SVT). Which characteristic should the nurse know about SVT? a. Embolization to lungs may result in death. b. Clot may extend to deeper veins if untreated. c. Vein is tender to pressure and there is edema. d. Typically found in the iliac, inferior, or superior vena cava.

b. If left untreated, a superficial vein thrombosis (SVT) may extend to deeper veins and VTE may occur. VTE may embolize to the lungs and have tenderness to pressure and edema. SVTs usually occur in superficial leg veins and have tenderness, itchiness, redness, warmth, pain, inflammation, and induration along the course of the superficial vein.

5. What is the pathophysiologic mechanism that results in the pulmonary edema of left-sided heart failure? a. Increased right ventricular preload b. Increased pulmonary hydrostatic pressure c. Impaired alveolar oxygen and carbon dioxide exchange d. Increased lymphatic flow of pulmonary extravascular fluid

b. In left-sided heart failure, blood backs up into the pulmonary veins and capillaries. This increased hydrostatic pressure in the vessels causes fluid to move out of the vessels and into the pulmonary interstitial space. When increased lymphatic flow cannot remove enough fluid from the interstitial space, fluid moves into the alveoli, resulting in pulmonary edema and impaired alveolar oxygen and carbon dioxide exchange. Initially the right side of the heart is not involved.

13. A patient with a small AAA is not a good surgical candidate. What should the nurse teach the patient is one of the best ways to prevent expansion of the lesion? a. Avoid strenuous physical exertion. b. Control hypertension with prescribed therapy. c. Comply with prescribed anticoagulant therapy. d. Maintain a low-calcium diet to prevent calcification of the vessel.

b. Increased systolic blood pressure (SBP) continually puts pressure on the diseased area of the artery, promoting its expansion. Small aneurysms can be treated by decreasing blood pressure (BP), modifying atherosclerosis risk factors, and monitoring the size of the aneurysm. Anticoagulants are used during surgical treatment of aneurysms but physical activity is not known to increase their size. Calcium intake is not related to calcification in arteries.

40. The patient has hypertension and just experienced an MI. Which type of medication would be expected to be added to decrease the workload on his heart? a. ACE inhibitor c. Calcium channel blocker b. β-adrenergic blocker d. Angiotensin II receptor blocker (ARB)

b. It is recommended that patients with hypertension and after an MI be on β-adrenergic blockers indefinitely to decrease oxygen demand. They inhibit sympathetic nervous stimulation of the heart; reduce heart rate, contractility, and blood pressure; and decrease afterload. Although calcium channel blockers decrease heart rate, contractility, and blood pressure, they are not used unless the patient cannot tolerate β-adrenergic blockers. ACE inhibitors and angiotensin II receptor blockers (ARBs) are used for vasodilation.

5. When teaching the patient with PAD about modifying risk factors associated with the condition, what should the nurse emphasize? a. Amputation is the ultimate outcome if the patient does not alter lifestyle behaviors. b. Modifications will reduce the risk of other atherosclerotic conditions such as stroke. c. Risk-reducing behaviors initiated after angioplasty can stop the progression of the disease. d. Maintenance of normal body weight is the most important factor in controlling arterial disease.

b. PAD occurs as a result of atherosclerosis and the risk factors are the same as for other diseases associated with atherosclerosis, such as CAD, cerebrovascular disease, and aneurysms. Major risk factors are tobacco use, hyperlipidemia, elevated C-reactive protein, diabetes mellitus, and uncontrolled hypertension. The risk for amputation is high in patients with severe occlusive disease but this is not the best approach to encourage patients to make lifestyle modifications.

24. Which drugs would the nurse expect to be included in those prescribed for patients with a mechanical valve replacement? a. Oral nitrates b. Anticoagulants c. Atrial antidysrhythmics d. β-adrenergic blocking agents

b. Patients with mechanical valves have an increased risk for thrombus formation. Therefore prophylactic anticoagulation therapy is used to prevent thrombus formation and systemic or pulmonary embolization. Nitrates are contraindicated for the patient with aortic stenosis because an adequate preload is necessary to open the stiffened aortic valve. Antidysrhythmics are used only if dysrhythmias occur and β-adrenergic blocking drugs may be used to control the heart rate if needed.

12. The patient with acute pericarditis is having a pericardiocentesis. Postoperatively what complication should the nurse monitor the patient for? a. Pneumonia b. Pneumothorax c. Myocardial infarction (MI) d. Cerebrovascular accident (CVA)

b. Pneumothorax may occur as a needle is inserted into the pericardial space to remove fluid for analysis and relieve cardiac pressure with pericardiocentesis. Other complications could include dysrhythmias, further cardiac tamponade, myocardial laceration, and coronary artery laceration.

14. Priority Decision: A patient with stage 2 hypertension who is taking hydrochlorothiazide (HydroDiuril) and lisinopril (Prinivil) has prazosin (Minipress) added to the medication regimen. What is most important for the nurse to teach the patient to do? a. Weigh every morning to monitor for fluid retention b. Change position slowly and avoid prolonged standing c. Use sugarless gum or candy to help relieve dry mouth d. Take the pulse daily to note any slowing of the heart rate

b. Prazosin is an α-adrenergic blocker that causes dilation of arterioles and veins and causes orthostatic hypotension. The patient may feel dizzy, weak, and faint when assuming an upright position after sitting or lying down and should be taught to change positions slowly, avoid standing for long periods, do leg exercises to increase venous return, and lie or sit down when dizziness occurs. Direct-acting vasodilators often cause fluid retention; dry mouth occurs with diuretic use, although orthostatic hypotension may occur with hydrochlorothiazide as well; and centrally acting α- and β-blockers may cause bradycardia.

46. In counseling the patient about sexual activity following an MI, what should the nurse do? a. Wait for the patient to ask about resuming sexual activity b. Discuss sexual activity while teaching about other physical activity c. Have the patient ask the health care provider when sexual activity can be resumed d. Inform the patient that impotence is a common long-term complication following MI

b. Resumption of sexual activity is often difficult for patients to approach and it is reported that most cardiac patients do not resume sexual activity after MI. The nurse can give the patient permission to discuss concerns about sexual activity by introducing it as a physical activity when other physical activities are discussed. Health care providers may have preferences regarding the timing of resumption of sexual activity and the nurse should discuss this with the health care provider and the patient but addressing the patient's concerns is a nursing responsibility. Patients should be informed that impotence after MI is common but that it usually disappears after several attempts.

17. During the patient's acute postoperative period following repair of an AAA, the nurse should ensure that which goal is achieved? a. Hypothermia is maintained to decrease oxygen need. b. BP and all peripheral pulses are evaluated at least every hour. c. IV fluids are administered at a rate to maintain urine output of 100 mL/hr. d. The patient's BP is kept lower than baseline to prevent leaking at the incision line.

b. The BP and peripheral pulses are evaluated every hour in the acute postoperative period to ensure that BP is adequate and that extremities are being perfused. BP is kept within normal range. If BP is too low, thrombosis of the graft may occur; if it is too high, it may cause leaking or rupture at the suture line. Hypothermia is induced during surgery but the patient is rewarmed as soon as surgery is completed. Fluid replacement to maintain urine output at 100 mL/hr would increase the BP too much and only 30 mL/hr of urine is needed to show adequate renal perfusion.

20. Priority Decision: A patient who is postoperative following repair of an AAA has been receiving IV fluids at 125 mL/hr continuously for the last 12 hours. Urine output for the last 4 hours has been 60 mL, 42 mL, 28 mL, and 20 mL, respectively. What is the priority action that the nurse should take? a. Monitor for a couple more hours. b. Contact the physician and report the decrease in urine output. c. Send blood for electrolytes, blood urea nitrogen (BUN), and creatinine. d. Decrease the rate of infusion to prevent blood leakage at the suture line.

b. The decreasing urine output is evidence that either the patient needs volume or there is reduced renal blood flow. The physician will want to be notified as soon as possible of this change in condition and may order laboratory tests. The other options are incorrect.

2. What accurately describes the pathophysiology of CAD? a. Partial or total occlusion of the coronary artery occurs during the stage of raised fibrous plaque b. Endothelial alteration may be caused by chemical irritants such as hyperlipidemia or by tobacco use c. Collateral circulation in the coronary circulation is more likely to be present in the young patient with CAD d. The leading theory of atherogenesis proposes that infection and fatty dietary intake are the basic underlying causes of atherosclerosis

b. The etiology of CAD includes atherosclerosis as the major cause. The pathophysiology of atherosclerosis development is related to endothelial chemical injury and inflammation, which can be the result of tobacco use, hyperlipidemia, hypertension, toxins, diabetes mellitus, hyperhomocysteinemia, and infection causing a local inflammatory response in the inner lining of the vessel walls. Partial or total occlusion occurs in the complicated lesion stage. Extra collateral circulation occurs in the presence of chronic ischemia. Therefore it is more likely to occur in an older patient.

10. A patient with acute pericarditis has markedly distended jugular veins, decreased BP, tachycardia, tachypnea, and muffled heart sounds. The nurse recognizes that these symptoms occur when what happens? a. The pericardial space is obliterated with scar tissue and thickened pericardium b. Excess pericardial fluid compresses the heart and prevents adequate diastolic filling c. The parietal and visceral pericardial membranes adhere to each other, preventing normal myocardial contraction d. Fibrin accumulation on the visceral pericardium infiltrates into the myocardium, creating generalized myocardial dysfunction

b. The patient is experiencing a cardiac tamponade that consists of excess fluid in the pericardial sac, which compresses the heart and the adjoining structures, preventing normal filling and cardiac output. Fibrin accumulation, a scarred and thickened pericardium, and adherent pericardial membranes occur in chronic constrictive pericarditis.

3. While obtaining patient histories, which patient does the nurse identify as having the highest risk for CAD? a. A white man, age 54, who is a smoker and has a stressful lifestyle b. A white woman, age 68, with a BP of 172/100 mm Hg and who is physically inactive c. An Asian woman, age 45, with a cholesterol level of 240 mg/dL and a BP of 130/74 mm Hg d. An obese African American man, age 65, with a cholesterol level of 195 mg/dL and a BP of 128/76 mm Hg

b. This white woman has one unmodifiable risk factor (age) and two major modifiable risk factors (hypertension and physical inactivity). Her gender risk is as high as a man's because she is over 65 years of age. The white man has one unmodifiable risk factor (gender), one major modifiable risk factor (smoking), and one minor modifiable risk factor (stressful lifestyle). The Asian woman has only one major modifiable risk factor (hyperlipidemia) and Asians in the United States have fewer myocardial infarctions (MIs) than do whites. The African American man has an unmodifiable risk factor related to age and one major modifiable risk factor (obesity).

9. A patient with chronic heart failure has atrial fibrillation and a left ventricular ejection fraction (LVEF) of 18%. To decrease the risk of complications from these conditions, what drug does the nurse anticipate giving? a. Diuretics c. β-Adrenergic blockers b. Anticoagulants d. Potassium supplements

b. Thrombus formation occurs in the heart when the chambers do not contract normally and empty completely. Both atrial fibrillation and very low left ventricular output (LVEF <20%) lead to thrombus formation, which is treated with anticoagulants to prevent the release of emboli into the circulation as well as antidysrhythmics or cardioversion to control atrial fibrillation.

33. Delegation Decision: In planning care for a patient who has just returned to the unit following a PCI, the nurse may delegate which activity to unlicensed assistive personnel (UAP)? a. Monitor the IV fluids and measure urine output. b. Check vital signs and report changes in HR, BP, or pulse oximetry. c. Explain to the patient the need for frequent vital signs and pulse checks. d. Assess circulation to the extremity used by checking pulses, skin temperature, and color.

b. Unlicensed assistive personnel (UAP) can check vital signs and report results to the RN. The other actions include assessment, teaching, and monitoring of IV fluids, which are all responsibilities of the RN.

19. Delegation Decision: The unit is very busy and short staffed. What could be delegated to the unlicensed assistive personnel (UAP)? a. Administer antihypertensive medications to stable patients. b. Obtain orthostatic blood pressure (BP) readings for older patients. c. Check BP readings for the patient receiving IV enalapril (Vasotec). d. Teach about home BP monitoring and use of automatic BP monitoring equipment.

b. Unlicensed assistive personnel (UAP) may check postural changes in BP as directed. The licensed practical nurse (LPN) may administer antihypertensive medications to stable patients. The RN must monitor the patient receiving IV enalapril (Vasotec), as he or she is in a hypertensive crisis. The RN must also do the teaching related to home BP monitoring.

14. When obtaining a nursing history for a patient with myocarditis, what should the nurse specifically question the patient about? a. Prior use of digoxin for treatment of cardiac problems b. Recent symptoms of a viral illness, such as fever and malaise c. A history of coronary artery disease (CAD) with or without an MI d. A recent streptococcal infection requiring treatment with penicillin

b. Viruses are the most common cause of myocarditis in the United States and early manifestations of myocarditis are often those of systemic viral infections. Myocarditis may also be associated with systemic inflammatory and metabolic disorders as well as with other microorganisms, drugs, or toxins. The patient with myocarditis is predisposed to drug-related dysrhythmias and toxicity with digoxin, so it is used very cautiously, if at all, in treatment of the condition.

18. A patient with rheumatic heart disease with carditis asks the nurse how long his activity will be restricted. What is the best answer by the nurse? a. "Full activity will be allowed as soon as acute symptoms have subsided." b. "Bed rest will be continued until symptoms of heart failure are controlled." c. "Nonstrenuous activities can be performed as soon as antibiotics are started." d. "Bed rest must be maintained until antiinflammatory therapy has been discontinued."

b. When carditis is present in the patient with rheumatic fever, ambulation is postponed until any symptoms of heart failure are controlled with treatment and full activity cannot be resumed until antiinflammatory therapy has been discontinued. In the patient without cardiac involvement, ambulation may be permitted as soon as acute symptoms have subsided and normal activity can be resumed when antiinflammatory therapy is discontinued.

12. What are manifestations of acute coronary syndrome (ACS) (select all that apply)? a. Dysrhythmia b. Stable angina c. Unstable angina d. ST-segment-elevation myocardial infarction (STEMI) e. Non-ST-segment-elevation myocardial infarction (NSTEMI)

c, d, e. Unstable angina, ST-segment-elevation myocardial infarction (STEMI), and non-ST-segment-elevation myocardial infarction (NSTEMI) are conditions that are manifestations of acute coronary syndrome (ACS). The other options are not manifestations of ACS.

15. What can be auscultated in a patient with cardiac valve problems (select all that apply)? a. Arterial bruit b. Pulsus alternans c. Cardiac murmurs d. Third heart sound (S3) e. Pericardial friction rub f. Fourth heart sound (S4)

c, d, f. The cardiac murmurs are produced by turbulent blood flow across diseased heart valves, S3 is heard with mitral valve regurgitation, and S4 is heard with aortic stenosis. Arterial bruits are from turbulent peripheral blood flow. Pulsus alternans, seen in heart failure, is a variation in the strength of each pulse when palpated. Pericardial friction rub is the sound heard with pericarditis.

3. Which arteries are the major providers of coronary circulation (select all that apply)? a. Left marginal artery b. Right marginal artery c. Left circumflex artery d. Right coronary artery e. Posterior descending artery f. Left anterior descending artery

c, d, f. The left circumflex and left anterior descending arteries branch from the left coronary artery. The left coronary artery and right coronary artery arise from the aorta to supply the atria, ventricles, and interventricular septum.

15. What types of angina can occur in the absence of CAD (select all that apply)? a. Silent ischemia d. Microvascular angina b. Nocturnal angina e. Chronic stable angina c. Prinzmetal's angina

c, d. Prinzmetal's angina and microvascular angina may occur in the absence of CAD but with arterial spasm in Prinzmetal's angina or abnormalities of the coronary microcirculation. Silent ischemia is prevalent in persons with diabetes mellitus and contributes to asymptomatic myocardial ischemia. Nocturnal angina occurs only at night. Chronic stable angina refers to chest pain that occurs with the same pattern of onset, duration, and intensity intermittently over a long period of time.

30. A second 12-lead ECG performed on a patient 4 hours after the onset of chest pain reveals ST segment elevation. What does the nurse recognize that this finding indicates? a. Transient ischemia typical of unstable angina b. Lack of permanent damage to myocardial cells c. MI associated with prolonged and complete coronary thrombosis d. MI associated with transient or incomplete coronary artery occlusion

c. A differentiation is made between MIs that have ST segment elevations on ECG and those that do not because chest pain accompanied by ST segment elevations is associated with prolonged and complete coronary thrombosis and is treated with reperfusion therapy. The other options are incorrect.

18. A 2400-mg sodium diet is prescribed for a patient with chronic heart failure. The nurse recognizes that additional teaching is necessary when the patient makes which statement? a. "I should limit my milk intake to 2 cups a day." b. "I can eat fresh fruits and vegetables without worrying about sodium content." c. "I can eat most foods as long as I do not add salt when cooking or at the table." d. "I need to read the labels on prepared foods and medicines for their sodium content."

c. All foods that are high in sodium should be eliminated in a 2400-mg sodium diet, in addition to the elimination of salt during cooking. Examples include obviously salted snack foods as well as pickles, processed prepared foods, and many sauces and condiments.

11. A surgical repair is planned for a patient who has a 5.5-cm abdominal aortic aneurysm (AAA). On physical assessment of the patient, what should the nurse expect to find? a. Hoarseness and dysphagia b. Severe back pain with flank ecchymosis c. Presence of a bruit in the periumbilical area d. Weakness in the lower extremities progressing to paraplegia

c. Although most abdominal aortic aneurysms (AAAs) are asymptomatic, on physical examination a pulsatile mass in the periumbilical area slightly to the left of the midline may be detected and bruits may be audible with a stethoscope placed over the aneurysm. Hoarseness and dysphagia may occur with aneurysms of the ascending aorta and the aortic arch. Severe back pain with flank ecchymosis is usually present on rupture of an AAA and neurovascular loss in the lower extremities may occur from pressure of a thoracic aneurysm.

21. When caring for a patient after a cardiac catheterization with coronary angiography, which finding would be of most concern to the nurse? a. Swelling at the catheter insertion site b. Development of raised wheals on the patient's trunk c. Absence of pulses distal to the catheter insertion site d. Patient pain at the insertion site as 4 on a scale of 0 to 10

c. An absence of pulses distal to the catheter insertion site indicates that clotting is occluding blood flow to the extremity and is an emergency that requires immediate medical attention. Some swelling and pain at the site are expected but the site is also monitored for bleeding and a pressure dressing and perhaps a sandbag or clamp may be applied. Hives may occur as a result of iodine sensitivity and will require treatment but the priority is the lack of pulses.

12. Which classification of drugs used to treat hypertension prevents the action of angiotensin II and promotes increased salt and water excretion? a. Thiazide diuretics c. Angiotensin II receptor blockers (ARBs) b. Direct vasodilators d. Angiotensin-converting enzyme (ACE) inhibitors

c. Angiotensin II receptor blockers (ARBs) prevent the action of angiotensin II and produce vasodilation and increased salt and water excretion. Thiazide diuretics decrease extracellular fluid volume by increasing Na+ and Cl- excretion with water. Direct vasodilators act directly on smooth muscle of arterioles to cause vasodilation. Angiotensin-converting enzyme (ACE) inhibitors prevent the conversion of angiotensin I to angiotensin II.

26. At what point in the healing process of the myocardium following an infarct does early scar tissue result in an unstable heart wall? a. 2 to 3 days after MI c. 10 to 14 days after MI b. 4 to 10 days after MI d. 6 weeks after MI

c. At 10 to 14 days after MI, the myocardium is considered especially vulnerable to increased stress because of the unstable state of healing at this point, as well as the increasing physical activity of the patient. At 2 to 3 days, removal of necrotic tissue is taking place by phagocytic cells. By 4 to 10 days, the necrotic tissue has been cleared and a collagen matrix for scar tissue has been deposited. Healing with scar-tissue replacement of the necrotic area is usually complete by 6 weeks.

10. Which diagnostic test is most useful in differentiating dyspnea related to pulmonary effects of heart failure from dyspnea related to pulmonary disease? a. Exercise stress testing c. B-type natriuretic peptide (BNP) levels b. Cardiac catheterization d. Determination of blood urea nitrogen (BUN)

c. B-type natriuretic peptide (BNP) is released from the ventricles in response to increased blood volume in the heart and is a good marker for heart failure. If BNP is elevated, shortness of breath is due to heart failure; if BNP is normal, dyspnea is due to pulmonary disease. BNP opposes the actions of the renin-angiotensin-aldosterone system, resulting in vasodilation and reduction in blood volume. Exercise stress testing and cardiac catheterization are more important tests to diagnose coronary artery disease and although the blood urea nitrogen (BUN) may be elevated in heart failure, it is a reflection of decreased renal perfusion. (See Table 32-6.)

35. Collaborative care of the patient with NSTEMI differs from that of a patient with STEMI in that NSTEMI is more frequently initially treated with what? a. PCI c. Acute intensive drug therapy b. CABG d. Reperfusion therapy with thrombolytics

c. Because an NSTEMI is an acute coronary syndrome that indicates a transient thrombosis or incomplete coronary artery occlusion, treatment involves intensive drug therapy with antiplatelets, glycoprotein IIb/IIIa inhibitors, antithrombotics, and heparin to prevent clot extension. In addition, IV nitroglycerin is used. Reperfusion therapy using thrombolytics, CABG, or PCI is used for treatment of STEMI.

3. What describes the action of the natriuretic peptides and nitric oxide in their counterregulatory processes in response to heart failure (HF)? a. Excretion of potassium c. Vasodilation and decreased blood pressure b. Increased release of ADH d. Decreased glomerular filtration rate and edema

c. Both the natriuretic peptides and nitric oxide contribute to vasodilation, decreased blood pressure, and decreased afterload. The natriuretic peptides also increase excretion of sodium by increasing glomerular filtration rate and diuresis (renal effects) as well as interfere with ADH release and inhibit aldosterone and renin secretion (hormonal effects).

11. What is the primary BP effect of β-adrenergic blockers such as atenolol (Tenormin)? a. Vasodilation of arterioles by blocking movement of calcium into cells b. Decrease Na+ and water reabsorption by blocking the effect of aldosterone c. Decrease CO by decreasing rate and strength of the heart and renin secretion by the kidneys d. Vasodilation caused by inhibiting sympathetic outflow from the central nervous system (CNS)

c. Cardioselective β-adrenergic blockers decrease CO, reduce sympathetic vasoconstrictor tone, and decrease renin secretion by kidneys. Calcium channel blockers reduce BP by causing blocking movement of calcium into cells, which causes vasodilation of arterioles. Spironolactone blocks the effect of aldosterone. Central adrenergic antagonists such as clonidine (Catapres) inhibit sympathetic outflow from the central nervous system (CNS).

36. During treatment with reteplase (Retavase) for a patient with a STEMI, which finding should most concern the nurse? a. Oozing of blood from the IV site b. BP of 102/60 mm Hg with an HR of 78 bpm c. Decrease in the responsiveness of the patient d. Presence of intermittent accelerated idioventricular dysrhythmias

c. Decreasing level of consciousness (LOC) may reflect hypoxemia resulting from internal bleeding, which is always a risk with thrombolytic therapy. Oozing of blood is expected, as are reperfusion dysrhythmias. BP is low but not considered abnormal because the pulse is within normal range.

11. A 62-year-old woman has prehypertension (BP 142/90 mm Hg) and smokes a pack of cigarettes per day. She has no symptoms of CAD but a recent LDL level was 154 mg/dL (3.98 mmol/L). Based on these findings, the nurse would expect that which treatment plan would be used first for this patient? a. Diet and drug therapy c. Diet therapy and smoking cessation b. Exercise instruction only d. Drug therapy and smoking cessation

c. Diet therapy and smoking cessation are indicated for a patient without CAD who has prehypertension and an LDL level ≥130 mg/dL. When the patient's LDL level is ≥160 mg/dL, drug therapy would be added to diet therapy. Because tobacco use is related to increased BP and LDL level, the benefit of smoking cessation is almost immediate. Exercise is indicated to reduce risk factors throughout treatment.

39. What is the rationale for using docusate sodium (Colace) for a patient after an MI? a. Controls ventricular dysrhythmias c. Minimizes bradycardia from vagal stimulation b. Relieves anxiety and cardiac workload d. Prevents the binding of fibrinogen to platelets

c. Docusate sodium (Colace) is a stool softener, which prevents straining and provoking dysrhythmias. It does not do any of the other options. Antidysrhythmics are used to control ventricular dysrhythmias; morphine sulfate is used to decrease cardiac workload and anxiety; and glycoprotein IIb/IIIa inhibitors and antiplatelets prevent the binding of fibrinogen to platelets.

13. The patient with chronic heart failure is being discharged with a diuretic, a renin-angiotensin-aldosterone system (RAAS) inhibitor, and a β-adrenergic blocker. When received from the pharmacy, which medication should not be included for this patient? a. Losartan (Cozaar) c. Dopamine (Intropin) b. Carvedilol (Coreg) d. Hydrochlorothiazide (HCTZ)

c. Dopamine (Intropin) is a β-adrenergic agonist that is a positive inotrope given IV, not orally, and used for acute HF. Losartan (Cozaar) is an angiotensin II receptor blocker used for patients who do not tolerate angiotensin-converting enzyme inhibitors. Carvedilol (Coreg) is the β-adrenergic blocker that blocks the sympathetic nervous system's negative effects on the failing heart. Hydrochlorothiazide (HCTZ) is the diuretic.

18. Priority Decision: Following an ascending aortic aneurysm repair, what is an important finding that the nurse should report immediately to the health care provider? a. Shallow respirations and poor coughing b. Decreased drainage from the chest tubes c. A change in level of consciousness (LOC) and inability to speak d. Lower extremity pulses that are decreased from the preoperative baseline

c. During repair of an AAA, the blood supply to the carotid arteries may be interrupted, leading to neurologic complications manifested by a decreased level of consciousness (LOC) and altered pupil responses to light as well as changes in facial symmetry, speech, and movement of the upper extremities. The thorax is opened for ascending aortic surgery and shallow breathing, poor cough, and decreasing chest drainage are expected. Often, lower limb pulses are normally decreased or absent for a short time following surgery.

32. Which treatment is used first for the patient with a confirmed MI to open the blocked artery within 90 minutes of arrival to the facility? a. Stent placement c. Percutaneous coronary intervention (PCI) b. Coronary artery bypass graft (CABG) d. Transmyocardial laser revascularization (TMR)

c. Emergent percutaneous coronary intervention (PCI) is the first treatment for patients with a confirmed MI within 90 minutes of arriving at the facility with an interventional cardiac catheterization lab. Stent placement, CABG, and TMR are usually done to facilitate circulation in non- emergency situations.

20. Which manifestation is an indication that a patient is having a hypertensive emergency? a. Symptoms of a stroke with an elevated BP b. A systolic BP >200 mm Hg and a diastolic BP >120 mm Hg c. A sudden rise in BP accompanied by neurologic impairment d. A severe elevation of BP that occurs over several days or weeks

c. Hypertensive emergency, a type of hypertensive crisis, is a situation that develops over hours or days in which a patient's BP is severely elevated with evidence of acute target organ disease (e.g., cerebrovascular, cardiovascular, renal, or retinal). The neurologic manifestations are often similar to the presentation of a stroke but do not show the focal or lateralizing symptoms of stroke. Hypertensive crises are defined by the degree of organ damage and how rapidly the BP rises, not by specific BP measurements. A hypertensive urgency is a less severe crisis in which a patient's BP becomes severely elevated over days or weeks but there is no evidence of target organ damage.

47. What advice about sexual activity should the nurse give to a male patient who has had an MI? a. The patient should use the superior position. b. Foreplay may cause too great an increase in heart rate. c. Prophylactic nitroglycerin may be used if angina occurs. d. Performance can be enhanced with the use of sildenafil (Viagra).

c. It is not uncommon for a patient who experiences chest pain on exertion to have some angina during sexual stimulation or intercourse and the patient should be instructed to use nitroglycerin prophylactically. Positionsduring intercourse are a matter of individual choice and foreplay is desirable because it allows a gradual increase in HR. Sildenafil (Viagra) should be used cautiously in men with CAD and should not be used with nitrates.

6. Priority Decision: During care of the patient following femoral bypass graft surgery, the nurse immediately notifies the health care provider if the patient experiences a. fever and redness at the incision site. b. 2+ edema of the extremity and pain at the incision site. c. a loss of palpable pulses and numbness and tingling of the feet. d. increasing ankle-brachial indices and serous drainage from the incision.

c. Loss of palpable pulses, numbness and tingling of the extremity, extremity pallor, cyanosis or cold, and decreasing ankle-brachial indices are indications of occlusion of the bypass graft and need immediate medical attention. Pain, redness, and serous drainage at the incision site are expected postoperatively.

28. In which patient would a mechanical prosthetic valve be preferred over a biologic valve for valve replacement? a. 41-year-old man with peptic ulcer disease b. 22-year-old woman who desires to have children c. 35-year-old man with a history of seasonal asthma d. 62-year-old woman with early Alzheimer's disease

c. Mechanical prosthetic valves require long-term anticoagulation and this is a factor in making a decision about the type of valve to use for replacement. Patients who cannot take anticoagulant therapy, such as women of childbearing age, patients at risk for hemorrhage, patients who may not be compliant with anticoagulation therapy, and patients over age 65 may be candidates for the less durable biologic valves.

21. What accurately describes mitral valve prolapse? a. Rapid onset prevents left chamber dilation b. May be caused by pulmonary hypertension c. Ballooning of valve into left atrium during ventricular systole d. Rapid development of pulmonary edema and cardiogenic shock

c. Mitral valve prolapse is the ballooning of the valve leaflets into the left atrium during ventricular systole. The rapid onset that prevents left chamber dilation and the rapid development of pulmonary edema and cardiogenic shock occur with acute mitral regurgitation. Pulmonary hypertension may contribute to tricuspid valve disease.

20. Delegation Decision: An RN is working with a licensed practical nurse (LPN) in caring for a group of patients on a cardiac telemetry unit. A patient with aortic stenosis has the nursing diagnosis of activity intolerance related to fatigue and exertional dyspnea. Which nursing activity could be delegated to the LPN? a. Explain the reason for planning frequent periods of rest. b. Evaluate the patient's understanding of his disease process. c. Monitor BP, HR, RR, and SpO2 before, during, and after ambulation. d. Teach the patient which activities to choose that will gradually increase endurance.

c. Monitoring vital signs before and after ambulation is the collection of data. Instructions should be provided to the licensed practical nurse (LPN) regarding what changes in these vital signs should be reported to the RN. The other actions listed are RN responsibilities.

12. Priority Decision: A patient is admitted to the emergency department with acute decompensated heart failure (ADHF). Which IV medication would the nurse expect to administer first? a. Digoxin (Lanoxin) c. Nesiritide (Natrecor) b. Morphine sulfate d. Bumetanide (Bumex)

c. Nesiritide (Natrecor) is a recombinant form of a natriuretic peptide that decreases preload and afterload by reducing pulmonary artery wedge pressure (PAWP) and systolic BP which decreases the workload of the heart for short-term emergency treatment of acute decompensated heart failure (ADHF). Digoxin (Lanoxin) requires a loading dose and time to work, so it is not recommended for emergency treatment of ADHF. Morphine sulfate relieves dyspnea but has more adverse events and mortality. Bumetanide (Bumex) will decrease fluid volume but also will decrease potassium levels and activate the sympathetic nervous system and renin-angiotensin-al

3. Following teaching about medications for PAD, the nurse determines that additional instruction is necessary when the patient makes which statement? a. "I should take one aspirin a day to prevent clotting in my legs." b. "The lisinopril I use for my blood pressure may help me walk further without pain." c. "I will need to have frequent blood tests to evaluate the effect of the Coumadin I will be taking." d. "Pletal should help me increase my walking distance and help prevent clots from forming in my legs."

c. Oral anticoagulants (warfarin) are not recommended for treatment of PAD but all of the other statements are correct in relation to treatment of PAD.

27. A patient is scheduled for an open surgical valvuloplasty of the mitral valve. In preparing the patient for surgery, what should the nurse know about this surgery? a. Cardiopulmonary bypass is not required with this procedure. b. Valve repair is a palliative measure, whereas valve replacement is curative. c. The operative mortality rate is lower in valve repair than in valve replacement. d. Patients with valve repair do not require postoperative anticoagulation as do those who have valve replacement.

c. Repair of mitral or tricuspid valves has a lower operative mortality rate than does replacement and is becoming the surgical procedure of choice for these valvular diseases. Open repair is more precise than closed repair and requires cardiopulmonary bypass during surgery. All types of valve surgery are palliative, not curative, and patients require lifelong health care. Anticoagulation therapy is used for all valve surgery for at least some time postoperatively.

5. How is secondary hypertension differentiated from primary hypertension? a. Has a more gradual onset than primary hypertension b. Does not cause the target organ damage that occurs with primary hypertension c. Has a specific cause, such as renal disease, that often can be treated by medicine or surgery d. Is caused by age-related changes in BP regulatory mechanisms in people over 65 years of age

c. Secondary hypertension has an underlying cause that can often be treated, in contrast to primary or essential hypertension, which has no single known cause. Isolated systolic hypertension occurs when the systolic blood pressure (SBP) is consistently elevated over 140 mm Hg but the diastolic blood pressure (DBP) remains less than 90 mm Hg, which is more prevalent in older adults. The only type of hypertension that does not cause target organ damage is pseudohypertension.

15. A 38-year-old man is treated for hypertension with triamterene and hydrochlorothiazide (Maxzide) and metoprolol (Lopressor). Four months after his last clinic visit, his BP returns to pretreatment levels and he admits he has not been taking his medication regularly. What is the nurse's best response to this patient? a. "Try always to take your medication when you carry out another daily routine so you do not forget to take it." b. "You probably would not need to take medications for hypertension if you would exercise more and stop smoking." c. "The drugs you are taking cause sexual dysfunction in many patients. Are you experiencing any problems in this area? d. "You need to remember that hypertension can be only controlled with medication, not cured, and you must always take your medication."

c. Sexual dysfunction, which can occur with many of the antihypertensive drugs, including thiazide and potassium- sparing diuretics and β-adrenergic blockers, can be a major reason that a male patient does not adhere to his treatment regimen. It is helpful for the nurse to raise the subject because sexual problems may be easier for the patient to discuss and handle once it has been explained that the drug might be the source of the problem.

20. The nurse determines that treatment of heart failure has been successful when the patient experiences a. weight loss and diuresis. b. warm skin and less fatigue. c. clear lung sounds and decreased HR. d. absence of chest pain and improved level of consciousness (LOC).

c. Successful treatment of heart failure is indicated by an absence of symptoms of pulmonary edema and hypoxemia, such as clear lung sounds and a normal HR. Weight loss and diuresis, warm skin, less fatigue, and improved LOC may occur without resolution of pulmonary symptoms. Chest pain is not a common finding in heart failure unless coronary artery perfusion is impaired.

29. Delegation Decision: The nursing care area is very busy with new surgical patients. Which care could the RN delegate to the unlicensed assistive personnel (UAP) for a patient with VTE? a. Assess the patient's use of herbs. b. Measure the patient for elastic compression stockings. c. Remind the patient to flex and extend the legs and feet every 2 hours d. Teach the patient to call emergency medical services (EMS) with signs of pulmonary embolus

c. The RN could delegate to the unlicensed assistive personnel (UAP) the task to remind the patient to flex and extend the legs and feet every 2 hours while in bed. Measuring for elastic compression stockings may be delegated to the licensed practical nurse (LPN). The RN must assess and teach the patient.

16. The nursing student is seeking assistance in hearing the patient's abnormal heart sounds. What should the nurse tell the student to do for a more effective assessment? a. Use the diaphragm of the stethoscope with the patient prone b. Use the diaphragm of the stethoscope with the patient supine c. Use the bell of the stethoscope with the patient leaning forward d. Use the bell of the stethoscope with the patient on the right side

c. The bell of the stethoscope will enable better hearing of the low-pitched extra heart sounds. Having the patient lean forward best enables hearing the aortic and pulmonic areas; having the patient on the left side will enhance the mitral area sounds; both of these positions bring the heart closer to the chest wall. Having the patient supine or prone will not improve the auscultation.

6. A patient with aortic valve endocarditis develops dyspnea, crackles in the lungs, and restlessness. What should the nurse suspect that the patient is experiencing? a. Pulmonary embolization from valve vegetations b. Vegetative embolization to the coronary arteries c. Valvular incompetence with resulting heart failure d. Nonspecific manifestations that accompany infectious diseases

c. The dyspnea, crackles, and restlessness that the patient is manifesting are symptoms of heart failure and decreased cardiac output (CO) that occurs in up to 80% of patients with aortic valve endocarditis as a result of aortic valve incompetence. Vegetative embolization from the aortic valve occurs throughout the arterial system and may affect any body organ. Pulmonary emboli occur in right-sided endocarditis.

1. Which patient is most likely to be in the fibrous stage of development of coronary artery disease (CAD)? a. Age 40, thrombus adhered to the coronary artery wall b. Age 50, rapid onset of disease with hypercholesterolemia c. Age 32, thickened coronary arterial walls with narrowed vessel lumen d. Age 19, elevated low-density lipoprotein (LDL) cholesterol, lipid-filled smooth muscle cells

c. The fibrous plaque stage has progressive changes that can be seen by age 30. Collagen covers the fatty streak and forms a fibrous plaque in the artery. The thrombus adheres to the arterial wall in the complicated lesion stage. Rapid onset of coronary artery disease (CAD) with hypercholesterolemia may be related to familial hypercholesterolemia, not a stage of CAD development. The fatty streak stage is the earliest stage of atherosclerosis and can be seen by age 15.

27. To detect and treat the most common complication of MI, what should the nurse do? a. Measure hourly urine output. c. Use continuous cardiac monitoring. b. Auscultate the chest for crackles. d. Take vital signs every 2 hours for the first 8 hours.

c. The most common complication of MI is cardiac dysrhythmias. Continuous cardiac monitoring allows identification and treatment of dysrhythmias that may cause further deterioration of the cardiovascular status or death. Measurement of hourly urine output and vital signs is indicated to detect symptoms of the complication of cardiogenic shock. Crackles, dyspnea, and tachycardia may indicate the onset of heart failure.

34. A patient is scheduled to have CABG surgery. What does the nurse explain to him that is involved with the procedure? a. A synthetic graft will be used as a tube for blood flow from the aorta to a coronary artery distal to an obstruction. b. A stenosed coronary artery will be resected and a synthetic arterial tube graft will be inserted to replace the diseased artery. c. The internal mammary artery will be detached from the chest wall and attached to a coronary artery distal to the stenosis. d. Reversed segments of a saphenous artery from the aorta will be anastomosed to the coronary artery distal to an obstruction.

c. The most common method of coronary artery bypass involves leaving the internal mammary artery attached to its origin from the subclavian artery but dissecting it from the chest wall and anastomosing it distal to an obstruction in a coronary artery. Synthetic grafts are not commonly used as coronary bypass grafts, although research continues to investigate this option. Saphenous veins are used for bypass grafts when additional conduits are needed.

22. During the nursing assessment of the patient with a distal descending aortic dissection, what should the nurse expect the patient to manifest? a. Altered LOC with dizziness and weak carotid pulses b. A cardiac murmur characteristic of aortic valve insufficiency c. Severe "ripping" back or abdominal pain with decreasing urine output d. Severe hypertension and orthopnea and dyspnea of pulmonary edema

c. The onset of an aortic dissection involving the distal descending aorta is usually characterized by a sudden, severe, tearing pain in the back; as it progresses down the aorta, the kidneys, abdominal organs, and lower extremities may begin to show evidence of ischemia. Aortic dissections of the ascending aorta and aortic arch may affect the heart and circulation to the head, with the development of cerebral ischemia, murmurs, ventricular failure, and pulmonary edema.

9. A patient is admitted to the hospital with a suspected acute pericarditis. To establish the presence of a pericardial friction rub, how should the nurse listen to the patient's chest? a. While timing the sound with the respiratory pattern b. With the bell of the stethoscope at the apex of the heart c. With the diaphragm of the stethoscope at the lower left sternal border of the chest d. With the diaphragm of the stethoscope to auscultate a high-pitched continuous rumbling sound

c. The stethoscope diaphragm at the left sternal border with the patient leaning forward is the best method to use to hear the high-pitched, grating sound of a pericardial friction rub. The sound does not radiate widely and occurs with the heartbeat. To differentiate a pericardial friction rub from a pleural friction rub, have the patient hold his or her breath. The rub will still be heard if it is cardiac in nature.

26. A patient is scheduled for a percutaneous transluminal balloon valvuloplasty. The nurse understands that this procedure is indicated for which patient? a. Any patient with aortic regurgitation b. Older patients with aortic regurgitation c. Older patients with stenosis of any valve d. Young adult patients with mild mitral valve stenosis

c. This procedure has been used for repair of mitral, tricuspid, and pulmonic stenosis and less often for aortic stenosis. It is usually used for older patients and for those patients who are poor surgical risks because it is relatively easy and has good results and few complications.

31. What describes transmyocardial laser revascularization (TMR)? a. Structure applied to hold vessels open b. Requires anticoagulation following the procedure c. Laser-created channels between left ventricular cavity and coronary circulation d. Surgical construction of new vessels to carry blood beyond obstructed coronary artery

c. Transmyocardial laser revascularization (TMR) is a treatment used for patients with inoperable CAD. It uses a high-energy laser to create channels in the heart to allow blood to flow to the ischemic area and can be done percutaneously or during surgery with a left anterior thoracotomy incision. A stent is the structure used to hold vessels open and requires anticoagulation following the procedure. Surgical construction of new vessels is done with a coronary artery bypass graft (CABG).

22. When a patient reports chest pain, why must unstable angina be identified and rapidly treated? a. The pain may be severe and disabling. b. ECG changes and dysrhythmias may occur during an attack. c. Atherosclerotic plaque deterioration may cause complete thrombus of the vessel lumen. d. Spasm of a major coronary artery may cause total occlusion of the vessel with progression to MI

c. Unstable angina is associated with the rupture of a once-stable atherosclerotic plaque, exposing the intima to blood and stimulating platelet aggregation and local vasoconstriction with thrombus formation. Patients with unstable angina require immediate hospitalization and monitoring because the lesion is at increased risk of complete thrombosis of the lumen with progression to MI. Any type of angina may be associated with severe pain, ECG changes, and dysrhythmias. Prinzmetal's angina is characterized by coronary artery spasm.

14. What causes the pain that occurs with myocardial ischemia? a. Death of myocardial tissue b. Dysrhythmias caused by cellular irritability c. Lactic acid accumulation during anaerobic metabolism d. Elevated pressure in the ventricles and pulmonary vessels

c. When the coronary arteries are occluded, contractility ceases after several minutes, depriving the myocardial cells of glucose and oxygen for aerobic metabolism. Anaerobic metabolism begins and lactic acid accumulates, irritating myocardial nerve fibers that then transmit a pain message to the cardiac nerves and upper thoracic posterior roots. The other factors may occur during vessel occlusion but are not the source of pain.

1. The nurse is administering a dose of digoxin (Lanoxin) to a patient with heart failure (HF). The nurse would become concerned with the possibility of digitalis toxicity if the patient reported which symptom(s)? A Muscle aches B Constipation C Pounding headache D Anorexia and nausea

d Anorexia, nausea, vomiting, blurred or yellow vision, and cardiac dysrhythmias are all signs of digitalis toxicity. The nurse would become concerned and notify the health care provider if the patient exhibited any of these symptoms.

12. A thoracic aortic aneurysm is found when a patient has a routine chest x-ray. The nurse anticipates that additional diagnostic testing to determine the size and structure of the aneurysm will include which test? a. Angiography b. Ultrasonography c. Echocardiography d. Computed tomography (CT) scan

d. A computed tomography (CT) scan is the most accurate test to determine the diameter of the aneurysm and whether a thrombus is present. The other tests may also be used but the CT scan yields the most descriptive results.

12. When palpating the patient's popliteal pulse, the nurse feels a vibration at the site. How should the nurse record this finding? a. Thready, weak pulse b. Bruit at the artery site c. Bounding pulse volume d. Thrill of the popliteal artery

d. A palpable vibration of a blood vessel is called a thrill and usually indicates turbulent blood flow through the vessel. A weak, thready pulse has little pressure and is difficult to palpate. A bruit is an abnormal buzzing or humming sound that may be auscultated over diseased blood vessels and a bounding pulse is an extra full, hard pulse that may occur with atherosclerosis or hypervolemia.

22. A female patient has a total cholesterol level of 232 mg/dL (6.0 mmol/L) and a high-density lipoprotein (HDL) of 65 mg/dL (1.68 mmol/L). A male patient has a total cholesterol level of 200 mg/dL and an HDL of 32 mg/dL. Based on these findings, which patient has the highest cardiac risk? a. The man, because his HDL is lower b. The woman, because her HDL is higher c. The woman, because her cholesterol is higher d. The man, because his cholesterol-to-HDL ratio is higher

d. A risk assessment for coronary artery disease (CAD) is determined by comparing the total cholesterol to high- density lipoprotein (HDL) and a ratio can be calculated by dividing the total cholesterol level by the HDL level. The ratio provides more information than either value alone and an increased ratio indicates an increased risk. The female patient has a ratio of 3.56, which is average risk, compared with the male patient's ratio of 6.56, which is increased risk.

22. What causes a sudden onset of cardiovascular collapse? a. Mitral stenosis b. Tricuspid valve disease c. Pulmonic valve stenosis d. Acute aortic regurgitation

d. Acute aortic regurgitation causes a sudden cardiovascular collapse. With mitral valve stenosis dyspnea is a prominent symptom and embolization may result from chronic atrial fibrillation. With tricuspid and pulmonic valve diseases, stenosis occurs more often than regurgitation. Tricuspid valve stenosis results in right atrial enlargement and elevated systemic venous pressures. Pulmonic valve stenosis results in right ventricular hypertension and hypertrophy.

19. Priority Decision: The nurse caring for a patient immediately following a transesophageal echocardiogram (TEE) should consider which action the highest priority? a. Monitor the ECG b. Monitor pulse oximetry c. Assess vital signs (BP, HR, RR, temperature) d. Maintain NPO status until gag reflex has returned

d. All actions will be done but in order to perform a transesophageal echocardiogram (TEE), the throat must be numbed. Until sensation returns, as evidenced by the gag reflex, the patient is at risk of aspiration so this action has the highest priority (priority related to airway—ABCs).

7. The laboratory tests for four patients show the following results. Which patient should the nurse teach first about preventing CAD because the patient is at the greatest risk for CAD even without other risk factors? a. Total cholesterol: 152 mg/dL, triglycerides: 148 mg/dL, LDL: 148 mg/dL, HDL: 52 mg/dL b. Total cholesterol: 160 mg/dL, triglycerides: 102 mg/dL, LDL: 138 mg/dL, HDL: 56 mg/dL c. Total cholesterol: 200 mg/dL, triglycerides: 150 mg/dL, LDL: 160 mg/dL, HDL: 48 mg/dL d. Total cholesterol: 250 mg/dL, triglycerides: 164 mg/dL, LDL: 172 mg/dL, HDL: 32 mg/dL

d. All of this patient's results are abnormal. The patient in option c is close to being at risk, as all of that patient's results are at or near the cutoff for being acceptable. If this patient is a woman, the HDL is too low. The other patients' results are at acceptable levels.

17. The nurse monitors the patient receiving treatment for acute decompensated heart failure with the knowledge that marked hypotension is most likely to occur with the IV administration of which medication? a. Furosemide (Lasix) c. Milrinone (Primacor) b. Nitroglycerin (Tridil) d. Nitroprusside (Nipride)

d. Although all of these drugs may cause hypotension, nitroprusside is a potent dilator of both arteries and veins and may cause such marked hypotension that an inotropic agent (e.g., dobutamine) administration may be necessary to maintain the BP during its administration. Furosemide may cause hypotension because of diuretic-induced depletion of intravascular fluid volume. Nitroglycerin is a vasodilator and can decrease BP but not as severely as nitroprusside. It primarily dilates veins and increases myocardial oxygen supply. Milrinone has a positive inotropic effect in addition to direct arterial dilation.

17. Tachycardia that is a response of the sympathetic nervous system to the pain of ischemia is detrimental because it increases oxygen demand and a. increases cardiac output. b. causes reflex hypotension. c. may lead to atrial dysrhythmias. d. impairs perfusion of the coronary arteries.

d. An increased heart rate (HR) decreases the time the heart spends in diastole, which is the time of greatest coronary blood flow. Unlike other arteries, coronary arteries are perfused when the myocardium relaxes and blood backflows from the aorta into the sinuses of Valsalva, which have openings to the right and left coronary arteries. Thus the heart has a decreased oxygen supply at a time when there is an increased oxygen demand. Tachycardia may also lead to ventricular dysrhythmia. The other options are incorrect.

4. Priority Decision: While teaching women about the risks and incidence of CAD, what does the nurse emphasize? a. Smoking is not as significant a risk factor for CAD in women as it is in men. b. Women seek treatment sooner than men when they have symptoms of CAD. c. Estrogen replacement therapy in postmenopausal women decreases the risk for CAD. d. CAD is the leading cause of death in women, with a higher mortality rate after MI than in men.

d. CAD is the number-one killer of American women and women have a much higher mortality rate within 1 year following MI than do men. Smoking carries specific problems for women because smoking has been linked to a decrease in estrogen levels and to early menopause and it has been identified as the most powerful contributor to CAD in women under the age of 50. Fewer women than men present with classic manifestations and women delay seeking care longer than men. Recent research indicates that estrogen replacement does not reduce the risk for CAD, even though estrogen lowers low-density lipoprotein (LDL) and raises high-density lipoprotein (HDL) cholesterol.

18. What is included in the correct technique for BP measurements? a. Always take the BP in both arms. b. Position the patient supine for all readings. c. Place the cuff loosely around the upper arm. d. Take readings at least two times at least 1 minute apart.

d. Correct technique in measuring BP includes taking two or more readings at least 1 minute apart. Initially BP measurements should be taken in both arms to detect any differences. If there is a difference, the arm with the higher reading should be used for all subsequent BP readings. The patient may be supine or sitting. The important point is that the arm being used is at the heart level and the cuff needs to fit snugly.

29. In the patient with chest pain, which results can differentiate unstable angina from an MI? a. ECG changes present at the onset of the pain b. A chest x-ray indicating left ventricular hypertrophy c. Appearance of troponin in the blood 48 hours after the infarct d. Creatine kinase (CK)-MB enzyme elevations that peak 18 hours after the infarct

d. Creatine kinase-muscle and brain subunits band (CK- MB) is a tissue enzyme that is specific to cardiac muscle and is released into the blood when myocardial cells die. CK-MB levels begin to rise about 6 hours after an acute MI, peak in about 18 hours, and return to normal within 24 to 36 hours. This increase can identify the presence of and quantify myocardiac damage. Cardiac troponin T and troponin I are released with myocardial damage, rise as quickly as CK-MB does, and remain elevated for 2 weeks. ECG changes are often not apparent immediately after infarct and may be normal when the patient seeks medical attention. An enlarged heart, determined by x-ray, indicates cardiac stress but is not diagnostic of acute MI.

19. Which observation made by the nurse should indicate the presence of the complication of graft thrombosis after aortic aneurysm repair? a. Cardiac dysrhythmias or chest pain b. Absent bowel sounds, abdominal distention, or diarrhea c. Increased temperature and increased white blood cell count d. Decreased pulses and cool, painful extremities below the level of repair

d. Decreased or absent pulses in conjunction with cool, painful extremities below the level of repair indicate graft thrombosis. Cardiac dysrhythmias or chest pain indicates myocardial ischemia. Absent bowel sounds, abdominal distention, diarrhea, or bloody stools indicate bowel infarction. Increased temperature and white blood cells, surgical site inflammation, or drainage indicates graft infection.

34. A patient with VTE is to be discharged on long-term warfarin (Coumadin) therapy and is taught about prevention and continuing treatment of VTE. The nurse determines that discharge teaching for the patient has been effective when the patient makes which statement? a. "I should expect that Coumadin will cause my stools to be somewhat black." b. "I should avoid all dark green and leafy vegetables while I am taking Coumadin." c. "Massaging my legs several times a day will help increase my venous circulation." d. "Swimming is a good activity to include in my exercise program to increase my circulation."

d. Exercise programs for patients recovering from VTE should emphasize swimming, which is particularly beneficial because of the gentle, even pressure of the water. Coumadin will not blacken stools. If this occurs, it could be a sign of gastrointestinal bleeding. Dark green and leafy vegetables have high amounts of vitamin K and should not be increased during Coumadin therapy but they do not need to be restricted. The legs must not be massaged because of the risk for dislodging any clots that may be present.

4. The acronym FACES is used to help educate patients to identify symptoms of heart failure. What does this acronym mean? a. Frequent activity leads to cough in the elderly and swelling b. Factors of risk: activity, cough, emotional upsets, salt intake c. Follow activity plan, continue exercise, and know signs of problems d. Fatigue, limitation of activities, chest congestion/cough, edema, shortness of breath

d. FACES is used to teach patients signs and symptoms of worsening heart failure. F = Fatigue; A = Activity limitations; C = Congestion/cough; E = Edema; S = Shortness of breath. The other options are not correct.

21. Which statement by the patient with chronic heart failure should cause the nurse to determine that additional discharge teaching is needed? a. "I will call my health clinic if I wake up breathless at night." b. "I will look for sodium content on labels of foods and over-the-counter medicines." c. "I plan to organize my household tasks so I don't have to constantly go up and down the stairs." d. "I should weigh myself every morning and go on a diet if I gain more than 2 or 3 pounds in 2 days."

d. Further teaching is needed if the patient believes a weight gain of 2 to 3 pounds in 2 days is an indication for dieting. In a patient with heart failure, this type of weight gain reflects fluid retention and is a sign of heart failure that should be reported to the health care provider. The other options show patient understanding of the heart failure management teaching. (See Table 35-10.)

13. Dietary teaching that includes dietary sources of potassium is indicated for the hypertensive patient taking which drug? a. Enalapril (Vasotec) c. Spironolactone (Aldactone) b. Labetalol (Normodyne) d. Hydrochlorothiazide (HydroDiuril)

d. Hydrochlorothiazide is a thiazide diuretic that causes sodium and potassium loss through the kidneys. High- potassium foods should be included in the diet or potassium supplements should be used to prevent hypokalemia. Enalapril and spironolactone may cause hyperkalemia by inhibiting the action of aldosterone and potassium supplements should not be used by patients taking these drugs. As a combined α/β-blocker, labetalol does not affect potassium levels.

21. Which drugs are most commonly used to treat hypertensive crises? a. Esmolol (Brevibloc) and captopril (Capoten) b. Enalaprilat (Vasotec) and minoxidil (Loniten) c. Labetalol (Normodyne) and bumetanide (Bumex) d. Fenoldopam (Corlopam) and sodium nitroprusside (Nipride)

d. Hypertensive crises are treated with IV administration of antihypertensive drugs, including the vasodilators sodium nitroprusside, fenoldopam, and nicardipine; adrenergic blockers such as phentolamine, labetalol, and esmolol; the ACE inhibitor IV enalaprilat; and the calcium channel blocker clevidipine. Sodium nitroprusside is the most effective parenteral drug for hypertensive emergencies. Drugs that are used specifically for hypertensive emergencies include sodium nitroprusside, nitroglycerin with myocardial infarction, hydralazine with other medications, and oral captopril.

23. What does the nursing responsibility in the management of the patient with hypertensive urgency often include? a. Monitoring hourly urine output for drug effectiveness b. Titrating IV drug dosages based on BP measurements every 2 to 3 minutes c. Providing continuous electrocardiographic (ECG) monitoring to detect side effects of the drugs d. Instructing the patient to follow up with a health care professional within 24 hours after outpatient treatment

d. Hypertensive urgencies are often treated with oral drugs on an outpatient basis but it is important for the patient to be seen by a health care professional within 24 hours to evaluate the effectiveness of the treatment. Hourly urine measurements, titration of IV drugs, and ECG monitoring are indicated for hypertensive emergencies.

15. Priority Decision: What is the most important role of the nurse in preventing rheumatic fever? a. Teach patients with infective endocarditis to adhere to antibiotic prophylaxis. b. Identify patients with valvular heart disease who are at risk for rheumatic fever. c. Encourage the use of antibiotics for treatment of all infections involving a sore throat. d. Promote the early diagnosis and immediate treatment of group A streptococcal pharyngitis.

d. Initial attacks of rheumatic fever and the development of rheumatic heart disease can be prevented by adequate treatment of group A streptococcal pharyngitis. Because streptococcal infection accounts for only about 20% of acute pharyngitis, cultures should be done to identify the organism and direct antibiotic therapy. Viral infections should not be treated with antibiotics. Prophylactic therapy is indicated in those who have valvular heart disease or have had rheumatic heart disease.

11. Which medication is currently approved only for use with African American patients for hypertension and angina? a. Captopril (Capoten) c. Spironolactone (Aldactone) b. Nitroglycerin (Nitro-Bid) d. Isosorbide dinitrate and hydralazine (BiDil)

d. Isosorbide dinitrate and hydralazine (BiDil) is currently used only with African American patients for hypertension and angina. Captopril (Capoten) is used only for hypertension by all patients. Nitroglycerin (Nitro-Bid) is used with hydralazine (Apresoline) for patients who cannot tolerate renin-angiotensin-aldosterone system inhibitors for heart failure management. Spironolactone (Aldactone) is used for hypertension.

4. What describes Janeway's lesions that are manifestations of infective endocarditis? a. Hemorrhagic retinal lesions b. Black longitudinal streaks in nail beds c. Painful red or purple lesions on fingers or toes d. Flat, red, painless spots on the palms of hands and soles of feet

d. Janeway's lesions are flat, painless, small red spots found on the palms of the hands and the soles of the feet. Black streaks on the nails are splinter hemorrhages. Hemorrhagic retinal lesions are Roth's spots. Painful lesions on the fingers and toes are Osler's nodes.

31. Which indirect thrombin inhibitor is only administered subcutaneously and does not need routine coagulation tests? a. Warfarin (Coumadin) b. Unfractionated heparin (Heparin) c. Hirudin derivatives (lepirudin [Refludan]) d. Low-molecular-weight heparin (nadroparin [Fraxiparine])

d. Low-molecular-weight heparin (nadroparin [Fraxiparine]) is only given subcutaneously and does not require routine coagulation testing. Unfractionated heparin is the only other indirect thrombin inhibitor option but it can be given subcutaneously or IV and therapeutic effects must be monitored with coagulation testing.

7. The nurse assesses the patient with chronic biventricular heart failure for paroxysmal nocturnal dyspnea (PND) by questioning the patient regarding a. the presence of difficulty breathing at night. b. frequent awakening to void during the night. c. the presence of a dry, hacking cough when resting. d. the use of two or more pillows to help breathing during sleep.

d. Paroxysmal nocturnal dyspnea (PND) is awakening from sleep with a feeling of suffocation and a need to sit up to be able to breathe and patients learn that sleeping with the upper body elevated on several pillows helps to prevent PND. Orthopnea is an inability to breathe effectively when lying down and nocturia occurs with heart failure as fluid moves back into the vascular system during recumbency, increasing renal blood flow.

21. Following discharge teaching with a male patient with an AAA repair, the nurse determines that further instruction is needed when the patient makes which statement? a. "I should avoid heavy lifting." b. "I may have some sexual dysfunction as a result of the surgery." c. "I should maintain a low-fat and low-cholesterol diet to help keep the new graft open." d. "I should take the pulses in my extremities and let the doctor know if they get too fast or too slow."

d. Patients are taught to palpate peripheral pulses to identify changes in their quality or strength but the rate is not a significant factor in peripheral perfusion. The color and temperature of the extremities are also important for patients to observe. The remaining statements are all true.

30. To help prevent embolization of the thrombus in a patient with a VTE, what should the nurse teach the patient to do? a. Dangle the feet over the edge of the bed q2-3hr. b. Ambulate around the bed three to four times a day. c. Keep the affected leg elevated above the level of the heart. d. Maintain bed rest until edema is relieved and anticoagulation is established.

d. Prevention of emboli formation can be achieved by bed rest and limiting movement of the involved extremity until the clot is stable, inflammation has receded, and anticoagulation is achieved. Dangling the legs promotes venous stasis and further clot formation and elevating the affected limb will promote venous return but it does not prevent embolization.

8. When instructing a patient with endocarditis how to prevent recurrence of the infection, what should the nurse teach the patient? a. Start on antibiotic therapy when exposed to persons with infections. b. Take one aspirin a day to prevent vegetative lesions from forming around the valves. c. Always maintain continuous antibiotic therapy to prevent the development of any systemic infection. d. Obtain prophylactic antibiotic therapy before certain invasive medical or dental procedures (e.g., dental cleaning).

d. Prophylactic antibiotic therapy should be initiated before invasive dental, medical, or surgical procedures to prevent recurrence of endocarditis. Continuous antibiotic therapy is indicated only in patients with implanted devices or ongoing invasive procedures. Symptoms of infection should be treated promptly but antibiotics are not used for exposure to infection.

10. What is a significant finding in the health history of a patient during an assessment of the cardiovascular system? a. Metastatic cancer b. Frequent viral pharyngitis c. Calcium supplementation d. Frequent use of recreational drugs

d. Recreational or abused drugs, especially stimulants such as cocaine and methamphetamine, are a growing cause of cardiac dysrhythmias and problems associated with tachycardia and IV injection of abused drugs is a risk factor for inflammatory and infectious conditions of the heart. Although calcium is involved in the contraction of muscles, calcium supplementation is not a significant factor in heart disease, nor is metastatic cancer. Streptococcal, but not viral, pharyngitis is a risk factor for rheumatic heart disease.

1. When obtaining a health history from a 72-year-old man with peripheral arterial disease (PAD) of the lower extremities, the nurse asks about a history of related conditions, including a. venous thrombosis. b. venous stasis ulcers. c. pulmonary embolism. d. coronary artery disease (CAD).

d. Regardless of the location, atherosclerosis is responsible for peripheral arterial disease (PAD) and is related to other cardiovascular disease and its risk factors, such as coronary artery disease (CAD) and carotid artery disease. Venous thrombosis, venous stasis ulcers, and pulmonary embolism are diseases of the veins and are not related to atherosclerosis.

13. Priority Decision: A patient with acute pericarditis has a nursing diagnosis of pain related to pericardial inflammation. What is the best nursing intervention for the patient? a. Administer opioids as prescribed on an around-the-clock schedule. b. Promote progressive relaxation exercises with the use of deep, slow breathing. c. Position the patient on the right side with the head of the bed elevated 15 degrees. d. Position the patient in Fowler's position with a padded over-the-bed table for the patient to lean on.

d. Relief from pericardial pain is often obtained by sitting up and leaning forward. Pain is increased by lying flat. The pain has a sharp, pleuritic quality that changes with respiration and patients take shallow breaths. Antiinflammatory medications may also be used to help control pain but opioids are not usually indicated.

22. The evaluation team for cardiac transplantation is evaluating patients. Which patient is most likely to receive the most benefit from a new heart? a. A 24-year-old man with Down syndrome who has received excellent care from parents in their 60s b. A 46-year-old single woman with a limited support system who has alcohol-induced cardiomyopathy c. A 60-year-old man with inoperable coronary artery disease who has not been compliant with lifestyle changes and rehabilitation programs d. A 52-year-old woman with end-stage coronary artery disease who has limited financial resources but is emotionally stable and has strong social support

d. The 52-year-old woman does not have any contraindications for cardiac transplantation, even though she lacks the indication of adequate financial resources. The postoperative transplant regimen is complex and rigorous and patients who have not been compliant with other treatments or who might not have the means to understand the care would not be good candidates. A history of drug or alcohol abuse is usually a contraindication to heart transplant.

10. Which aneurysm is uniform in shape and a circumferential dilation of the artery? a. False aneurysm b. Pseudoaneurysm c. Saccular aneurysm d. Fusiform aneurysm

d. The fusiform aneurysm is circumferential and relatively uniform in shape. The false aneurysm or pseudoaneurysm is not an aneurysm but a disruption of all of the arterial wall layers with bleeding that is contained by surrounding anatomic structures. Saccular aneurysms are the pouchlike bulge of an artery.

29. When performing discharge teaching for the patient following a mechanical valve replacement, the nurse determines that further instruction is needed when the patient says which statement? a. "I may begin an exercise program to gradually increase my cardiac tolerance." b. "I will always need to have my blood checked once a month for its clotting function." c. "I should take prophylactic antibiotics before I have dental or invasive medical procedures." d. "The biggest risk I have during invasive health procedures is bleeding because of my anticoagulants."

d. The greatest risk to a patient who has an artificial valve is the development of endocarditis with invasive medical or dental procedures. Before any of these procedures, antibiotic prophylaxis is necessary to prevent infection. Planning of an exercise program and monitoring anticoagulant therapy will be done.

7. Priority Decision: A patient hospitalized for 1 week with subacute infective endocarditis is afebrile and has no signs of heart damage. Discharge with outpatient antibiotic therapy is planned. During discharge planning with the patient, what is it most important for the nurse to do? a. Plan how his needs will be met while he continues on bed rest. b. Encourage the use of diversional activities to relieve boredom and restlessness. c. Teach the patient to avoid crowds and exposure to upper respiratory infections. d. Assess the patient's home environment in terms of family assistance and hospital access.

d. The patient with outpatient antibiotic therapy requires vigilant home nursing care and it is most important to determine the adequacy of the home environment for successful management of the patient. The patient is at risk for life-threatening complications, such as embolization and pulmonary edema, and must be able to access a hospital if needed. Bed rest will not be necessary for the patient without heart damage. Avoiding infections and planning diversional activities are indicated for the patient but are not the most important factors while he is on outpatient antibiotic therapy.

19. What is an effect of valvular regurgitation? a. It causes a pressure gradient difference across an open valve. b. A pericardial friction rub is heard on the right sternal border of the chest. c. It leads to decreased flow of blood and hypertrophy of the preceding chamber. d. There is a backward flow of blood and volume overload in the preceding chamber.

d. Valvular regurgitation causes a backward flow of blood and volume overload in the preceding chamber. Without treatment, eventually hypertrophy of that chamber occurs. Stenosis causes a pressure gradient difference and decreased blood flow and hypertrophy of the preceding chamber. A pericardial friction rub is not related to valvular regurgitation but would be heard at the lower left sternal border of the chest.


Kaugnay na mga set ng pag-aaral

CHAPTER 18: CARE OF PATIENTS WITH ARTHRITIS AND OTHER CONNECTIVE TISSUE DISEASES

View Set

Consumer Behavior - Chapter 14 - Exam 2

View Set

Childrens' Day greetings for you!

View Set

Prep U Chapter 41 Musculoskeletal

View Set

Gettier, "Is Justified True Belief Knowledge?"

View Set